critical and urgent care ii answers to self-assessment - accp

36
PSAP-VII • Critical and Urgent Care 1 Answers Fungal Infections in the Intensive Care Unit Questions 1 and 2 pertain to the following case. J.G. is a 42-year-old man who was in a motor vehicle acci- dent and suffered penetrating abdominal trauma 4 days ago. Immediately aſter the accident, he underwent an emer- gency splenectomy and repair of a fractured leſt femur. On postoperative day 3, J.G. remains in the surgical intensive care unit (ICU). He is receiving norepinephrine for blood pressure support and mechanical ventilation as well as piperacillin/tazobactam for empiric antibacterial coverage. He is currently afebrile but is expected to be in the ICU for at least a week. 1. Which diagnostic test would detect the most likely organism(s) to cause fungal infection in J.G.? A. Beta-glucan testing. B. Galactomannan testing. C. Fungal sputum cultures. D. Fungal polymerase chain reaction testing. 1. Answer: A Beta-glucan testing is able to detect both Candida and Aspergillus spp.; therefore, it would be a potentially useful screening tool for invasive fungal infections in the intensive care unit (ICU) seing (Answer A is correct). Experience from ICU patients suggests that a single measurement of beta-glucan is not useful in detecting disease; however, persistent elevation with several measurements can cor- rectly identify patients with invasive fungal infection. Galactomannan testing is inappropriate for this patient for many reasons, including no significant risk factors for invasive aspergillosis, which is the only fungus detected by galactomannan testing (Answer B is incorrect). Candida spp. are the more likely fungal pathogens in such a patient. In addition, this patient is receiving piperacillin/tazobactam, which has been associated with false-positive results with galactomannan. Culturing for fungi and other organisms should be performed as clinically indicated; however, pos- itive sputum cultures for Candida spp. usually represent colonization, and colonization can be difficult to differenti- ate from true lung disease (Answer C is incorrect). Finally, there is no commercially available and validated fungal polymerase chain reaction (PCR) assay and no study docu- menting the use of PCR as a prospective surveillance tool (Answer D is incorrect). 1. Presterl E, Parschalk B, Bauer E, Lassnigg A, Hajdu S, Graninger W. Invasive fungal infections and (1,3)-β-d-glucan serum con- centrations in long-term intensive care patients. Int J Infect Dis 2009;13:707–12. [PubMed Link] 2. Viscoli C, Machei M, Cappellano P, Bucci B, Bruzzi P, Van Lint MT, et al. False-positive galactomannan platelia Aspergillus test results for patients receiving piperacillin-tazobactam. Clin Infect Dis 2004;38:913–6. [PubMed Link] 3. el-Ebiary M, Torres A, Fabregas N, de la Bellacasa JP, González J, Ramirez J, et al. Significance of the isolation of Candida spe- cies from respiratory samples in critically ill, non-neutropenic patients. An immediate postmortem histologic study. Am J Respir Crit Care Med 1997;156:583–90. [PubMed Link] 2. Which of the following is the best evidence-based therapy for J.G.? A. Fluconazole 400 mg intravenously every 24 hours. B. Fluconazole 200 mg by nasogastric tube every 24 hours. C. Caspofungin 70 mg intravenously now; then 50 mg intravenously every 24 hours. D. No antifungal therapy. 2. Answer: D Based on available evidence, initiation of antifungal ther- apy for this is not warranted at this time (Answer D is correct). In one study, enteral fluconazole prophylaxis Critical and Urgent Care II Answers to Self-Assessment Questions

Upload: others

Post on 09-Feb-2022

3 views

Category:

Documents


0 download

TRANSCRIPT

PSAP-VII • Critical and Urgent Care 1 Answers

Fungal Infections in the Intensive Care Unit

Questions 1 and 2 pertain to the following case.J.G. is a 42-year-old man who was in a motor vehicle acci-dent and suffered penetrating abdominal trauma 4 days ago. Immediately after the accident, he underwent an emer-gency splenectomy and repair of a fractured left femur. On postoperative day 3, J.G. remains in the surgical intensive care unit (ICU). He is receiving norepinephrine for blood pressure support and mechanical ventilation as well as piperacillin/tazobactam for empiric antibacterial coverage. He is currently afebrile but is expected to be in the ICU for at least a week.

1. Which diagnostic test would detect the most likely organism(s) to cause fungal infection in J.G.?

A. Beta-glucan testing.B. Galactomannan testing.C. Fungal sputum cultures.D. Fungal polymerase chain reaction testing.

1. Answer: ABeta-glucan testing is able to detect both Candida and Aspergillus spp.; therefore, it would be a potentially useful screening tool for invasive fungal infections in the intensive care unit (ICU) setting (Answer A is correct). Experience from ICU patients suggests that a single measurement of beta-glucan is not useful in detecting disease; however, persistent elevation with several measurements can cor-rectly identify patients with invasive fungal infection. Galactomannan testing is inappropriate for this patient for many reasons, including no significant risk factors for invasive aspergillosis, which is the only fungus detected by galactomannan testing (Answer B is incorrect). Candida spp. are the more likely fungal pathogens in such a patient. In addition, this patient is receiving piperacillin/tazobactam, which has been associated with false-positive results with galactomannan. Culturing for fungi and other organisms

should be performed as clinically indicated; however, pos-itive sputum cultures for Candida spp. usually represent colonization, and colonization can be difficult to differenti-ate from true lung disease (Answer C is incorrect). Finally, there is no commercially available and validated fungal polymerase chain reaction (PCR) assay and no study docu-menting the use of PCR as a prospective surveillance tool (Answer D is incorrect).1. Presterl E, Parschalk B, Bauer E, Lassnigg A, Hajdu S, Graninger

W. Invasive fungal infections and (1,3)-β-d-glucan serum con-centrations in long-term intensive care patients. Int J Infect Dis 2009;13:707–12.

[PubMed Link]2. Viscoli C, Machetti M, Cappellano P, Bucci B, Bruzzi P, Van Lint

MT, et al. False-positive galactomannan platelia Aspergillus test results for patients receiving piperacillin-tazobactam. Clin Infect Dis 2004;38:913–6.

[PubMed Link]3. el-Ebiary M, Torres A, Fabregas N, de la Bellacasa JP, González

J, Ramirez J, et al. Significance of the isolation of Candida spe-cies from respiratory samples in critically ill, non-neutropenic patients. An immediate postmortem histologic study. Am J Respir Crit Care Med 1997;156:583–90.

[PubMed Link]

2. Which of the following is the best evidence-based therapy for J.G.?

A. Fluconazole 400 mg intravenously every 24 hours.

B. Fluconazole 200 mg by nasogastric tube every 24 hours.

C. Caspofungin 70 mg intravenously now; then 50 mg intravenously every 24 hours.

D. No antifungal therapy.

2. Answer: DBased on available evidence, initiation of antifungal ther-apy for this is not warranted at this time (Answer D is correct). In one study, enteral fluconazole prophylaxis

Critical and Urgent Care II

Answers to Self-Assessment Questions

PSAP-VII • Critical and Urgent Care2Answers

in patients who remained in the ICU longer than 3 days resulted in fewer fungal infections; however, there was no effect on overall duration of ICU stay or other important outcomes. The results generated interest in further defin-ing appropriate high-risk ICU patients who might develop invasive fungal infections. In addition, many clinicians are concerned with the other negative consequences that may occur from administering fluconazole prophylaxis to a large number of ICU patients (number needed to treat = 15), including the risk of developing azole-resistant Candida spp. (Answer A and Answer B are incorrect). To date, no studies have been completed that evaluate echinocandins as early preemptive therapy (Answer C is incorrect). In fur-ther attempts to define risk in ICU patients, the presence of a central venous catheter and receipt of systemic antibacte-rials were both important risk factors but were insufficient to predict subsequent fungal disease without other fac-tors. High risk is defined as the presence of at least two of the following: parenteral nutrition, dialysis, major surgery, pancreatitis, and use of corticosteroids or other immuno-suppressants. Attempts to evaluate this rule prospectively for initiating various antifungal therapies have not been suc-cessful. Therefore, although the patient in the case clearly has a central line and has received systemic antimicrobials in the past 3 days, he lacks a second additional risk factor to meet the definition of high risk. The best course of action is to initiate no antifungal therapy and continue to assess the patient carefully.1. Pelz RK, Hendrix CW, Swoboda SM, Diener-West M, Merz

WG, Hammond J, et al. Double-blind, placebo-controlled trial of fluconazole to prevent candidal infections in critically ill sur-gical patients. Ann Surg 2001;233:542–8.

[PubMed Link]2. Ostrosky-Zeichner L, Sable C, Sobel J, Alexander BD,

Donowitz G, Kan V, et al. Multicenter retrospective develop-ment and validation of a clinical prediction rule for nosocomial invasive candidiasis in the intensive care setting. Eur J Clin Microbiol Infect Dis 2007;26:271–6.

[PubMed Link]

3. As the critical care pharmacist at your institution, you have been asked by the ICU about the best applica-tion of the new fungal diagnostic technologies in your tertiary care medical center. Your particular unit has epidemiology data available, and you know that, for candidemia occurring on your unit, 45% are due to Candida albicans (65% susceptible to fluconazole); 30% due to Candida glabrata; and 15% due to Candida parapsilosis, with the rest due to other Candida spp. Which of the following is the most evidence-based response? A. Peptide nucleic acid fluorescence in situ

hybridization (PNA FISH) test.B. Serum galactomannan.

C. Beta-glucan.D. Both beta-glucan and galactomannan tests.

3. Answer: AData from pharmacoeconomic models as well as retrospec-tive experience from one institution have shown substantial reductions in antifungal drug expenditures when peptide nucleic acid fluorescence in situ hybridization (PNA FISH) was implemented as part of the routine diagnostic criteria for candidemia (Answer A is correct). A role for other new tests, including the beta-glucan assay, has yet to be proved in the ICU setting. Recent experiences have suggested that the beta-glucan assay, which tests for a variety of fungi, is elevated in ICU patients because of causes other than inva-sive fungal disease (Answer C and Answer D are incorrect). Therefore, using this test as a single diagnostic procedure is not appropriate. Beta-glucan may have use as part of rou-tine serial screening because persistent elevations seem more closely associated with disease. The role of galacto-mannan (Answer B and Answer D are incorrect) has been most clearly established as a sequential monitoring tool in patients with cancer. Given the relatively low incidence of invasive aspergillosis in the ICU in patients without under-lying immunosuppression, it is not appropriate to begin routine monitoring of galactomannan in ICU patients with-out further evidence to support this practice.1. Forrest GN, Mankes K, Jabra-Rizk MA, Weekes E, Johnson

JK, Lincalis DP, et al. Peptide nucleic acid fluorescence in situ hybridization-based identification of Candida albicans and its impact on mortality and antifungal therapy costs. J Clin Microbiol 2006;44:3381–3.

[PubMed Link]2. Alexander BD, Dodds Ashley ES, Reller LB, Reed SD. C o s t

savings with implementation of PNA FISH testing for identifi-cation of Candida albicans in blood cultures. Diagn Microbiol Infect Dis 2006;54:277–82.

[PubMed Link]3. Presterl E, Parschalk B, Bauer E, Lassnigg A, Hajdu S, Graninger

W. Invasive fungal infections and (1,3)-β-d-glucan serum con-centrations in long-term intensive care patients. Int J Infect Dis 2009;13:707–12.

[PubMed Link]

Questions 4–6 pertain to the following case. T.A. is a 36-year-old woman who has undergone hematopoietic stem cell transplantation for multiple myeloma. It has been 36 days since the transplant; she has had delayed engraftment, and her current absolute neutrophil count is 150 cells/mm3. She has been transferred to the medical ICU for treatment of sepsis with a systolic blood pressure in the 50s. She has received 3 L of fluid resuscitation. Before transfer to the ICU, she was receiving the following anti-infective and immunosuppressive medications: voriconazole 200 mg orally every 12 hours,

PSAP-VII • Critical and Urgent Care 3 Answers

tacrolimus 1 mg orally every 24 hours, and ciprofloxacin 500 mg orally every 12 hours.

4. Which of the following fungal or mold pathogens is the most likely etiologic agent of T.A.’s blood stream infection?A. C. glabrata.B. C. albicans.C. Aspergillus fumigatus.D. Zygomycetes spp.

4. Answer: AAlthough each of the listed pathogens has been associated with breakthrough infections in patients receiving voricon-azole, Candida spp. remain the most common fungal cause of septic episodes in ICU patients. In patients receiving voriconazole prophylaxis, Candida glabrata is commonly reported as a breakthrough pathogen, likely because of the decreased susceptibility of C. glabrata to fluconazole and other azole agents (Answer A is correct). C. albicans is a pos-sible pathogen, but it is not reported as often as C. glabrata as a cause of breakthrough infection in patients receiving vori-conazole prophylaxis (Answer B is incorrect). In addition, although associated with serious infections, clinical presen-tation of Aspergillus fumigatus and the Zygomycetes spp. does not typically mimic sepsis because these organisms are not associated with bloodstream infections (Answer C and Answer D are incorrect). 1. Vincent JL, Sakr Y, Sprung CL, Ranieri VM, Reinhart K,

Gerlach H, et al. Sepsis in European intensive care units: results of the SOAP study. Crit Care Med 2006;34:344–53.

[PubMed Link]2. Trifilio S, Singhal S, Williams S, Frankfurt O, Gordon L, Evens

A, et al. Breakthrough fungal infections after allogeneic hema-topoietic stem cell transplantation in patients on prophylactic voriconazole. Bone Marrow Transplant 2007;40:451–6.

[PubMed Link]

5. Which of the following treatments is best to recom-mend for T.A. for empiric antifungal coverage of molds? A. Voriconazole 200 mg orally every 12 hours.B. Voriconazole 4 mg/kg intravenously every 12

hours.C. Micafungin 50 mg intravenously every 24 hours.D. Amphotericin B lipid complex 5 mg/kg

intravenously every 24 hours.

5. Answer: DWhen patients experience a breakthrough fungal infection while receiving antifungal prophylaxis, it is recommended that empiric therapy be initiated with an agent from a different therapeutic class; therefore, neither of the vori-conazole options would be appropriate (Answer A and

Answer B are incorrect). Prophylactic voriconazole was being administered orally, and suboptimal concentrations with oral therapy have been associated with clinical failure. Without serum drug concentrations documenting a low or undetectable drug concentration or cultures suggesting a pathogen susceptible to voriconazole, the patient should be initiated on an antifungal agent with a different mechanism of action. Either micafungin or amphotericin B lipid com-plex would be a reasonable choice; however, the dosing of micafungin (50 mg/day) is the prophylactic dose for this agent; treatment with micafungin requires a dose of at least 100 mg/day (Answer C is incorrect). Therefore, amphoter-icin B lipid complex 5 mg/kg intravenously every 24 hours is the best therapeutic option given (Answer D is correct).1. Trifilio S, Ortiz R, Pennik G, Verma A, Pi J, Stosor V, et al.

Voriconazole therapeutic drug monitoring in allogeneic hematopoietic stem cell transplant recipients. Bone Marrow Transplant 2005;35:509–13.

[PubMed Link]2. Pappas PG, Rotstein CMF, Betts RF, Nucci M, Talwar D, De

Waele JJ, et al. Micafungin versus caspofungin for treatment of candidemia and other forms of invasive candidiasis. Clin Infect Dis 2007;45:883–93.

[PubMed Link]3. van Burik JA, Ratanatharathorn V, Stepan DE, Miller CB,

Lipton JH, Vesole DH, et al. Micafungin versus fluconazole for prophylaxis against invasive fungal infections during neu-tropenia in patients undergoing hematopoietic stem cell transplantation. Clin Infect Dis 2004;39:1407–16.

[PubMed Link]

6. Which of the following aspects of treatment for candidemia is associated with decreased mortality in T.A.?A. Initiation of antifungal therapy on the first day of

positive blood cultures.B. Negative blood assay for beta-glucan within the

first 24 hours of antifungal therapy.C. Removal of any central venous catheter within 48

hours of a positive blood culture.D. Initial therapy with a combination of an

echinocandin and an azole antifungal.

6. Answer: APrompt initiation of antifungal therapy is associated with a lower mortality (Answer A is correct). This finding is simi-lar to data obtained for other infections in the ICU setting. It has been proposed that beta-glucan would be an attractive tool to monitor response to therapy, but data are insuffi-cient to support its use at this time (Answer B is incorrect). Central venous catheters are the most commonly impli-cated source of candidemia, and many clinicians feel that they should be removed promptly; however, data from one of the micafungin candidemia trials did not show any

PSAP-VII • Critical and Urgent Care4Answers

difference in outcomes between patients who did or did not have their catheter removed. In addition, the recently updated Infectious Diseases Society of America (IDSA) guidelines for treatment of candidiasis state that there is insufficient evidence to recommend catheter removal for all patients with candidemia (Answer C is incorrect). Finally, there has been only one study assessing initial combination therapy with amphotericin B and fluconazole. Although the results of this study did favor the combination regimen, higher Acute Physiological and Chronic Health Evaluation II Scale (APACHE II) scores in patients enrolled in the flu-conazole monotherapy arm prevent firm conclusions based on these data, and patients in the combination arm did experience substantial amphotericin B–related toxicities (Answer D is incorrect). 1. Garey KW, Rege M, Pai MP, Mingo DE, Suda KJ, Turpin RS, et

al. Time to initiation of fluconazole therapy impacts mortality in patients with candidemia: a multi-institutional study. Clin Infect Dis 2006;43:25–31.

[PubMed Link]2. Pappas PG, Rotstein CMF, Betts RF, Nucci M, Talwar D, De

Waele JJ, et al. Micafungin versus caspofungin for treatment of candidemia and other forms of invasive candidiasis. Clin Infect Dis 2007;45:883–93.

[PubMed Link]3. Pappas PG, Kauffman CA, Andes D, Benjamin DK, Calandra

TF, Edwards JE, et al. Clinical practice guidelines for the man-agement of candidiasis: 2009 update by the Infectious Diseases Society of America. Clin Infect Dis 2009;48:503–35.

[PubMed Link]4. Rex JH, Pappas PG, Karchmer AW, Sobel J, Edwards JE, Hadley

S, et al. A randomized and blinded multicenter trial of high-dose fluconazole plus placebo versus fluconazole plus amphotericin B as therapy for candidemia and its consequences in nonneu-tropenic subjects. Clin Infect Dis 2003;36:1221–8.

[PubMed Link]

Questions 7 and 8 pertain to the following case. M.A., a 45-year-old man with diabetes mellitus and chronic kidney failure requiring dialysis, was admitted to the medical ICU 48 hours ago with diabetic ketoacidosis. He is febrile and hypotensive. An initial treatment regimen of piperacillin/tazobactam and vancomycin is initiated. Specimens from a recent bronchoalveolar lavage (BAL) with biopsy, blood, urine, sputum, and stool were sent to the microbiology laboratory for culture.

7. Identification of Candida spp. from which of the following sources is mostly likely to reflect true invasive infection and prompt initiation of antifun-gal therapy in M.A.? A. Urine.B. Sputum.C. Stool.D. BAL biopsy.

7. Answer: DIdentifying Candida spp. from the histopathology of a biopsy specimen is diagnostic for Candida pneumonia and should prompt treatment (Answer D is correct). Sputum cultures alone are not diagnostic for pneumonia and most likely indicate colonization (Answer B is incorrect). Urine cultures positive for Candida spp. also are often attribut-able to colonization and may not require treatment with antifungal drugs (Answer A is incorrect). The stool of up to 50% of patients in surgical ICUs is colonized with Candida, and this colonization does not indicate an active infection (Answer C is incorrect).1. Pappas PG, Kauffman CA, Andes D, Benjamin DK, Calandra

TF, Edwards JE, et al. Clinical practice guidelines for the man-agement of candidiasis: 2009 update by the Infectious Diseases Society of America. Clin Infect Dis 2009;48:503–35.

[PubMed Link]2. Barkauskas CE, Perfect JR. Candida pneumonia: what we know

and what we don’t. Curr Fungal Infect Rep 2009;3:21–31. [PubMed Link]3. Rangel-Frausto MS, Wiblin T, Blumberg HM, Saiman L,

Patterson J, Rinaldi M, et al. National epidemiology of mycoses survey (NEMIS): variations in rates of bloodstream infec-tions due to Candida species in seven surgical intensive care units and six neonatal intensive care units. Clin Infect Dis 1999;29:253–8.

[PubMed Link]

8. M.A.’s blood, urine, and sputum cultures were reported positive for yeast 24 hours after being drawn. Speciation is not yet available; however, according to the last antibiogram available at your institution, about 45% of Candida isolates are C. albicans, and fluconazole susceptibility for C. albicans is 98%. Which of the fol-lowing is the best empiric antifungal therapy for M.A.? A. Fluconazole 800 mg intravenously now; then 400

mg intravenously daily.B. Caspofungin 70 mg intravenously now; then 50

mg intravenously daily.C. Micafungin 50 mg intravenously daily.D. Amphotericin B lipid complex 5 mg/kg

intravenously daily.

8. Answer: BBased on rates of non-albicans candidemia of more than 50% and the severity of illness in this patient, initial empiric therapy should be targeted toward organisms with poten-tial resistance, intrinsic or acquired, to fluconazole (Answer A is incorrect). According to the IDSA guidelines, first-line treatment for this type of patient would consist of an appro-priately dosed echinocandin, such as caspofungin (Answer B is correct). Although micafungin would be an accept-able echinocandin, the proposed dose of 50 mg/day is the

PSAP-VII • Critical and Urgent Care 5 Answers

prophylactic dosage for hematopoietic stem cell transplan-tation recipients, a dose at which breakthrough candidal infections have occurred. It is also a dosage that has not been studied for treatment of candidemia (Answer C is incorrect). Amphotericin B is no longer considered first-line therapy in this setting (Answer D is incorrect).1. Pappas PG, Kauffman CA, Andes D, Benjamin DK, Calandra

TF, Edwards JE, et al. Clinical practice guidelines for the man-agement of candidiasis: 2009 update by the Infectious Diseases Society of America. Clin Infect Dis 2009;48:503–35.

[PubMed Link]2. vanBurik JA, Ratanatharathorn V, Stepan DE, Miller CB, Lipton

JH,Vesole DH, et al. Micafungin versus fluconazole for prophy-laxis against invasive fungal infections during neutropenia in patients undergoing hematopoietic stem cell transplantation. Clin Infect Dis 2004;39:1407–16.

[PubMed Link]3. Kuse ER, Chetchotisakd P, da Cunha CA, Ruhnke M, Barrios C,

Raghunadharao D, et al. Micafungin versus liposomal ampho-tericin B for candidaemia and invasive candidosis: a phase III randomised double-blind trial. Lancet 2007;369:1519–27.

[PubMed Link]

9. As the antimicrobial stewardship pharmacist reviewing the cultures from the surgical ICU, you notice a posi-tive blood culture for C. parapsilosis in a patient who recently suffered abdominal trauma. The patient has been receiving micafungin 100 mg intravenously for the past 2 days, has been clinically responding, has nor-mal kidney function, and is being prepared for transfer to the step-down unit later today. Which of the fol-lowing interventions is best at this time?A. Continue micafungin.B. Change therapy to amphotericin B lipid complex

5 mg/kg intravenously every 24 hours.C. Change therapy to fluconazole 400 mg orally

daily.D. Discontinue antifungal therapy because

the positive culture was likely caused by a contaminant.

9. Answer: CThe 2009 IDSA guidelines suggest that the preferred agent for a bloodstream infection caused by Candida parapsi-losis is fluconazole (Answer C is correct); the guidelines also state that patients receiving an echinocandin should be converted to fluconazole (Answer A is incorrect). This recommendation is made partly because of the higher min-imum inhibitory concentrations observed with isolates of C. parapsilosis and the echinocandins in early clinical tri-als. Amphotericin B preparations are traditionally reserved for refractory infections or as second-line treatment and are not indicated in this patient, who is clinically respond-ing to therapy (Answer B is incorrect). According to the 2009 IDSA guidelines, all fungemias should be treated for

2 weeks from the time of the first negative blood culture; therefore, discontinuing therapy is not an option at present (Answer D is incorrect).1. Pappas PG, Kauffman CA, Andes D, Benjamin DK, Calandra

TF, Edwards JE, et al. Clinical practice guidelines for the man-agement of candidiasis: 2009 update by the Infectious Diseases Society of America. Clin Infect Dis 2009;48:503–35.

[PubMed Link]2. Mora-Duarte J, Betts R, Rotstein C, Colombo AL, Thompson-

Moya L, Smietana J, et al. Comparison of caspofungin and amphotericin B for invasive candidiasis. N Engl J Med 2002;347:2020–9.

[PubMed Link]

10. A 24-year-old man with a penetrating abdominal gun-shot wound has been in the ICU for 7 days. He is intubated, his chest radiography today reveals question-able new infiltrates, and he has developed a low-grade fever. Sputum cultures are sent. He is initiated on cefepime for presumed hospital-acquired pneumonia, and his clinical condition remains unchanged 48 hours later. The initial sputum culture report is 2+ yeast. Which of the following is the best therapy recom-mendation for this patient? A. Initiate aerosolized liposomal amphotericin B

12.5 mg/day.B. Give caspofungin 70 mg intravenously now; then

50 mg intravenously daily.C. Initiate fluconazole 400 mg intravenously daily.D. Do not initiate antifungal therapy.

10. Answer: DManaging therapy for the ICU patient who has sputum cul-tures positive for yeast remains a clinical challenge. Available data suggest that for immunocompetent patients, this yeast is a colonizing organism. Without other evidence of dis-seminated or invasive disease (e.g., other positive cultures, histopathology supporting diagnosis), there is no need to provide treatment (Answer D is correct). Aerosolized amphotericin B has been used to prevent invasive fungal lung infection, but it has not shown efficacy to treat con-firmed disease (Answer A is incorrect). Either of the other two agents, caspofungin or fluconazole, would be appropri-ate if the patient had a confirmed infection (Answer B and Answer C are incorrect).1. Barkauskas CE, Perfect JR. Candida pneumonia: what we know

and what we don’t. Curr Fungal Infect Rep 2009;3:21–31. [PubMed Link]2. Rijnders BJ, Cornelissen JJ, Slobbe L, Becker MJ, Doorduijn

JK, Hop WCJ, et al. Aerosolized liposomal amphotericin B for the prevention of invasive pulmonary aspergillosis during pro-longed neutropenia: a randomized, placebo-controlled trial. Clin Infect Dis 2008;46:1401–8.

[PubMed Link]

PSAP-VII • Critical and Urgent Care6Answers

11. A 45-year-old man has been in the ICU for 5 days after orthotopic liver transplantation. His postopera-tive course has been complicated by intra-abdominal bleeding requiring two return trips to the operating room. His surveillance cultures obtained at the time of transplantation revealed a sputum culture positive for C. albicans and a single blood culture positive for coag-ulase- negative Staphylococcus. Which of the following is the best therapeutic approach for this patient?A. Initiate fluconazole 400 mg intravenously daily for

treatment of presumed candidal pneumonia.B. Initiate fluconazole 400 mg intravenously daily for

prophylaxis because this patient is at high risk of fungal infection.

C. Begin serial monitoring with beta-glucan testing and initiate fluconazole 400 mg intravenously daily if two sequential tests are positive.

D. Do not initiate antifungal therapy or testing at this time.

11. Answer: BThe best therapeutic approach in this high-risk patient at this time is to initiate fluconazole prophylaxis (Answer B is correct). In addition to his immunosuppression regimen, he has at least two other risk factors for invasive fungal infection after his liver transplantation. These risk factors include the need for reexploration because of intra-abdominal bleeding and the Candida colonization of his sputum. Treatment of Candida in the sputum is not clinically indicated at this time (Answer A is incorrect). Although sputum cultures posi-tive for Candida spp. in immunocompromised patients are cause for concern, in this man, the positive cultures more likely represent colonization; thus, no treatment is required. The culture was taken as surveillance before undergoing transplantation and before the patient was immunosup-pressed. Although the presence of Candida does represent an increased risk of invasive disease, it should be considered a risk factor but not an indication for treatment. Prospective monitoring for invasive fungal infection with beta-glucan is a promising concept but has not been validated in prospec-tive clinical trials (Answer C is incorrect). In addition, the presence of risk factors for invasive fungal infections in this patient and indications for prophylaxis make prophylaxis the most appropriate intervention rather than prospective monitoring or no action at all (Answer D is incorrect).1. Collins LA, Samore MH, Roberts MS, Luzzati R, Jenkins RL,

Lewis WD, et al. Risk factors for invasive fungal infections complicating orthotopic liver transplantation. J Infect Dis 1994;170:644–52.

[PubMed Link]2. American Thoracic Society; Infectious Diseases Society of

America. Guidelines for the management of adults with hos-pital-acquired, ventilator-associated, and healthcare-associated pneumonia. Am J Respir Crit Care Med 2005;171:388–416.

[PubMed Link]

Questions 12–15 pertain to the following case. J.D. is a 24-year-old man who received bilateral lung transplantation in 2006 for chronic obstructive pulmonary disease. He was hospitalized for treatment of cytomegalovirus infection, had an episode of respiratory distress, and required ICU admission. New, diffuse pulmonary nodules appear on computed tomography of the chest, and invasive aspergillosis is highly suspected. His current immunosuppression regimen consists of tacrolimus and prednisone. His other medications include ganciclovir, lorazepam, and pantoprazole.

12. Which of the following diagnostic tests will be most helpful in confirming a diagnosis of invasive asper-gillosis in J.D.?A. Galactomannan.B. Bronchoscopy with biopsy.C. Beta-glucan.D. PNA FISH.

12. Answer: BOf the answer choices given, only Answer B and Answer A would allow confirmation of invasive aspergillosis. The PNA FISH test is used for more rapid identification of C. albicans and C. glabrata, once a culture is identified as positive, but it does not aid in identifying Aspergillus spp. (Answer D is incorrect). Although the beta-glucan test is able to detect aspergillosis, it can also detect invasive fungal infections caused by many additional pathogens includ-ing Candida spp., Fusarium, Trichosporon, Saccharomyces, and Acremonium (Answer C is incorrect). Galactomannan is a useful test and is specific for the diagnosis of invasive aspergillosis, but its utility in the solid-organ transplant population is limited (Answer A is incorrect). In addition, galactomannan has been shown to best aid in the diagnosis of invasive aspergillosis as part of a prospective monitor-ing program, not as an isolated diagnostic test. Therefore, for this patient, the best confirmation of true invasive aspergillosis would be to perform bronchoscopy to obtain lung tissue for culture and histopathologic confirmation (Answer B is correct).1. Yoshida M, Obayashi T, Iwama A, Ito M, Tsunoda S, Suzuki T,

et al. Detection of plasma(1→3)-beta-d-glucan in patients with Fusarium, Trichosporon, Saccharomyces and Acremonium fungemias. J Med Vet Mycol 1997;35:371–4.

[PubMed Link]2. Husain S, Kwak EJ, Obman A, Wagener MM, Kusne S, Stout

JE, et al. Prospective assessment of Platelia Aspergillus galac-tomannan antigen for diagnosis of invasive aspergillosis in lung transplant recipients. Am J Transplant 2004;4:796–802.

[PubMed Link]

13. Therapy is initiated with broad-spectrum anti-bacterial therapy consisting of cefepime and tobramycin. Antifungal coverage includes voriconazole

PSAP-VII • Critical and Urgent Care 7 Answers

and caspofungin, with a loading dose of each being ordered. Within 5 minutes of beginning antimicrobial infusions, J.D. is flushed and experiencing some short-ness of breath. Which of the following is the most likely cause of this reaction? A. Symptoms of underlying lung infection.B. Histamine-mediated infusion reaction associated

with caspofungin.C. Allergic reaction to the cyclodextrin component

of voriconazole.D. A drug interaction between voriconazole and

lorazepam used for sedation.

13. Answer: BHistamine-mediated reactions are associated with all three of the available echinocandin antifungal agents (Answer B is correct). This reaction is related to the infusion rate and rapidly abates when the infusion is slowed or discontinued. Premedication before future doses can improve tolerabil-ity in patients who have this reaction. This reaction can be mistaken for a hypersensitivity reaction to the echinocan-din or to another component of the preparation or another medication being given, but these are rare (Answer C is incorrect). Voriconazole is known to interact with the ben-zodiazepines; however, this typically manifests as increased sedation caused by the benzodiazepine itself (Answer D is incorrect). Finally, the temporal relation to the drug admin-istration likely precludes symptoms of underling infection as a cause (Answer A is incorrect).1. Eschenauer G, DePestel DD, Carver PL. Comparison of echi-

nocandin antifungals. Ther Clin Risk Manag 2007;3:71–97. [PubMed Link]2. Cleary JD, Schwartz M, Rogers PD de Mestral J, Chapman

SW. Effects of amphotericin B and caspofungin on histamine expression. Pharmacotherapy 2003;23:966–73.

[PubMed Link]3. Saari TI, Laine K, Bertilsson L, Neuvonen PJ, Olkkola KT.

Voriconazole and fluconazole increase the exposure to oral diazepam. Eur J Clin Pharmacol 2007;63:941–9.

[PubMed Link]

14. It is now day 4 of combination antifungal therapy, but J.D. is clinically worsening. Antifungal therapy is changed to amphotericin B lipid complex 5 mg/kg intravenously every 24 hours. There is concern about nephrotoxicity given the concomitant aminoglycoside therapy. Which of the following strategies would best minimize the risk of kidney toxicity in J.D.? A. 0.9% sodium chloride 500 mL by infusion before

the amphotericin B infusion.B. Amphotericin B administered by continuous

infusion.

C. Acetaminophen 650 mg orally 30 minutes before the amphotericin infusion.

D. Diphenhydramine 50 mg orally 30 minutes before the amphotericin infusion.

14. Answer: AFew advances have been made in the management of amphotericin B toxicity in the past decade; however, strate-gies for addressing these toxicities may have been forgotten because the use of this agent has declined. Strategies to prevent infusion-related toxicities include premedication with acetaminophen (Answer C) and diphenhydramine (Answer D). However, saline loading in combination with adequate hydration effectively minimizes kidney toxic-ity associated with amphotericin B therapy and should be considered whenever nephrotoxicity is a concern (Answer A is correct; Answer C and Answer D are incorrect). More recently, prolonged infusion of amphotericin B has been proposed as a method to minimize kidney toxicity. This strategy decreases nephrotoxicity compared with tradi-tional 4-hour infusions. However, few patients with severe fungal infections have been studied (Answer B is incorrect). Furthermore, the pharmacodynamic properties of ampho-tericin B suggest concentration-dependent killing, leading to concerns of decreased efficacy with prolonged infusions (more than 4 hours).1. Gallis HA, Drew RH, Pickard WW. Amphotericin B: 30 years

of clinical experience. Rev Infect Dis 1990;12:308–29. [PubMed Link]2. Branch RA. Prevention of amphotericin B-induced renal

impairment. a review on the use of sodium supplementation. Arch Intern Med 1988;148:2389–94.

[PubMed Link]3. Peleg AY, Woods ML. Continuous and 4 h infusion of ampho-

tericin B: a comparative study involving high-risk haematology patients. J Antimicrob Chemother 2004;54:803–8.

[PubMed Link]

15. When the voriconazole and caspofungin loading doses were ordered for J.D., the transplant team appropriately adjusted all medications to account for drug-drug inter-actions with the antifungal regimen. Now that therapy is being changed, which of the following medica-tions is most likely to require a dosage adjustment?A. Tacrolimus.B. Lorazepam.C. Prednisone.D. Tobramycin.

15. Answer: AThe interaction between voriconazole and tacroli-mus has been well described. Recommendations to minimize adverse outcomes related to this interac-tion include an empiric tacrolimus dose reduction when

PSAP-VII • Critical and Urgent Care8Answers

initiating concomitant voriconazole therapy (Answer A is correct). This dose reduction should also be considered when an azole is removed from a tacrolimus-containing pharmacotherapy regimen. This patient’s tacrolimus dose should be returned to appropriate dosing when discontin-uing the voriconazole. Cases of rejection have occurred in patients suboptimally dosed on immunosuppressive agents after the removal of an interacting drug. Although both benzodiazepine and prednisone serum concentrations may be increased when voriconazole is added to the regimen, no recommendations exist for empiric dose reduction when initiating concomitant therapy with these agents (Answer B and Answer C are incorrect). There is no reported inter-action between voriconazole and tobramycin (Answer D is incorrect).1. Venkataramanan R, Zang S, Gayowski T, Singh N. Voriconazole

inhibition of the metabolism of tacrolimus in a liver transplant recipient and in human liver microsomes. Antimicrob Agents Chemother 2002;46:3091–3.

[PubMed Link]2. Saari TI, Laine K, Bertilsson L, Neuvonen PJ, Olkkola KT.

Voriconazole and fluconazole increase the exposure to oral diazepam. Eur J Clin Pharmacol 2007;63:941–9.

[PubMed Link]3. Saad AH, DePestel DD, Carver PL. Factors influencing the

magnitude and clinical significance of drug interactions between azole antifungals and select immunosuppressants. Pharmacotherapy 2006;26:1730–44.

[PubMed Link]

16. A new antifungal treatment algorithm for funge-mia has been implemented in the ICU. The protocol includes broad-spectrum empiric antifungal therapy, rapid identification of Candida spp. using PNA FISH, intravenous-to-oral therapy conversion guidelines, and defined treatment durations. You have been asked to identify appropriate metrics to track the success of this program from the institutional perspective. In which of the following is a reduction most likely to pro-vide substantial benefit to the institution? A. Echinocandin antifungals.B. ICU care by minimizing length of stayC. Renal replacement therapy for amphotericin B

toxicity.D. Surveillance cultures for Candida colonization.

16. Answer: ATwo published studies have assessed the economic impact of PNA FISH as part of a treatment algorithm. Both expe-riences documented important cost savings by sparing echinocandin use when C. albicans was identified at the same time as positive blood cultures for yeast (Answer A is correct). Although decreased length of stay in an ICU will save money, it has not been shown that using the PNA FISH

testing method will decrease length of stay (Answer B is incorrect). Few treatment guidelines for fungemia include the routine use of amphotericin B. Although the avoided costs for each case of nephrotoxicity can be considerable (estimated at $35,000 per case), current minimal con-ventional amphotericin B use will preclude considerable opportunities for cost savings (Answer C is incorrect). An algorithm for fungemia management is unlikely to address routine surveillance cultures in the ICU setting; therefore, it is unlikely to have a marked effect on these expenditures (Answer D is incorrect).1. Alexander BD, Dodds Ashley ES, Reller LB, Reed SD. C o s t

savings with implementation of PNA FISH testing for identifi-cation of Candida albicans in blood cultures. Diagn Microbiol Infect Dis 2006;54:277–82.

[PubMed Link]2. Forrest GN, Mankes K, Jabra-Rizk MA, Weekes E, Johnson

JK, Lincalis DP, et al. Peptic nucleic acid fluorescence in situ hybridization-based identification of Candida albicans and its impact on mortality and antifungal therapy costs. J Clin Microbiol 2006;44:3381–3.

[PubMed Link]3. Bates DW, Su L, Yu DT, Chertow GM, Seger DL, Gomes DRJ,

et al. Mortality and costs of acute renal failure associated with amphotericin B therapy. Clin Infect Dis 2001;32:686–93.

[PubMed Link]

17. A 23-year-old man receiving care in the burn-trauma ICU has been clinically stable but had a temperature of 99.5°F (37.5°C) on morning rounds. Surveillance cultures from sputum, urine (by the urinary catheter), and blood were obtained. The next morning, his urine culture is reported as positive for yeast, with more than 100,000 colony-forming units. Urinalysis did not reveal any important findings. Which of the following therapies is best for this patient? A. Remove the urinary catheter.B. Remove the urinary catheter and initiate

fluconazole 200 mg orally daily.C. Replace the urinary catheter and initiate

amphotericin B bladder irrigations.D. No action is needed.

17. Answer: ASelecting the appropriate therapy for an ICU patient with a urine culture positive for Candida is challenging. Expert consensus (2009 IDSA guidelines) regarding the treatment of invasive candidiasis suggests that in patients with no risk factors, the asymptomatic patient does not require treat-ment for funguria beyond removal of the urinary catheter (Answer A is correct). If a patient requires antifungal drug treatment, fluconazole remains the drug of choice (Answer B is incorrect). Amphotericin B bladder irrigation is an option, but its use has been associated with high relapse

PSAP-VII • Critical and Urgent Care 9 Answers

rates, and it is difficult to administer (Answer C is incor-rect). As previously stated, doing nothing is insufficient as long as the urinary catheter remains in place (Answer D is incorrect).1. Pappas PG, Kauffman CA, Andes D, Benjamin DK, Calandra

TF, Edwards JE, et al. Clinical practice guidelines for the man-agement of candidiasis: 2009 update by the Infectious Diseases Society of America. Clin Infect Dis 2009;48:503–35.

[PubMed Link]2. Jacobs LG, Skitmore EA, Cardoso LA, Ziv F. Bladder irrigation

with amphotericin B for treatment of fungal urinary tract infec-tions. Clin Infect Dis 1994;18:313–8.

[PubMed Link]3. Drew RH, Arthur RR, Perfect JR. Is it time to abandon the

use of amphotericin B bladder irrigation? Clin Infect Dis 2005;40:1465–70.

[PubMed Link]

Questions 18–20 pertain to the following case. M.M., a 36-year-old woman with Crohn’s disease, was admitted to the ICU with an enterocutaneous fistula. She has been receiving parenteral nutrition by a central venous catheter for the past 4 days. M.M. has severe sepsis and has systolic blood pressures in the 50s, pulse rates in the 130s, and a temperature of 102.7°F (39.3°C). Initial blood cultures are reported as positive for yeast.

18. Which of the following is the most likely causative fungal pathogen for M.M.’s sepsis?A. C. albicans.B. C. glabrata.C. C. parapsilosis.D. Candida lusitaniae.

18. Answer: CC. parapsilosis occurs more often in neonates, transplant recipients, and patients receiving parenteral nutrition. Therefore, this patient is at high risk of C. parapsilosis infec-tion (Answer C is correct). Each of the other pathogens, C. albicans, C. glabrata, and Candida lusitaniae, is possible but less likely given the use of parenteral nutrition in this patient (Answer A, Answer B, and Answer D are incorrect).1. Almirante B, Rodríguez D, Cuenca-Estrella M, Almela M,

Sanchez F, Ayats J, et al. Epidemiology, risk factors, and prognosis of Candida parapsilosis bloodstream infections: case-control population-based surveillance study of patients in Barcelona, Spain, from 2002 to 2003. J Clin Microbiol 2006;44:1681–5.

[PubMed Link]2. Pappas PG, Kauffman CA, Andes D, Benjamin DK, Calandra

TF, Edwards JE, et al. Clinical practice guidelines for the man-agement of candidiasis: 2009 update by the Infectious Diseases Society of America. Clin Infect Dis 2009;48:503–35.

[PubMed Link]

19. Which of the following initial empiric antifungal therapies provides the best coverage against all pos-sible pathogens in M.M. pending the availability of culture results? A. Anidulafungin 200 mg intravenously now; then

100 mg intravenously daily.B. Voriconazole 6 mg/kg intravenously every 12

hours for two doses; then 4 mg/kg intravenously every 12 hours.

C. Voriconazole 200 mg orally every 12 hours.D. Amphotericin B lipid complex 5 mg/kg

intravenously every 24 hours.

19. Answer: AThe echinocandins (e.g., anidulafungin) are the drugs of choice for initial treatment of candidemia in the critically ill patient such as the patient in this case (Answer A is cor-rect). Voriconazole has shown efficacy in the treatment of candidemia; however, its use offers no real advantage over initial therapy with fluconazole (Answer B and Answer C are incorrect). Caution is also warranted with adminis-tration of the intravenous formulation of voriconazole in patients with kidney insufficiency, which is commonly seen in hypotensive, septic patients. Finally, amphotericin B has largely been replaced as primary therapy for candidemia secondary to the available efficacy data with the echinocan-dins and associated toxicities with the polyene antifungal (Answer D is incorrect).1. Pappas PG, Kauffman CA, Andes D, Benjamin DK, Calandra

TF, Edwards JE, et al. Clinical practice guidelines for the man-agement of candidiasis: 2009 update by the Infectious Diseases Society of America. Clin Infect Dis 2009;48:503–35.

[PubMed Link]2. Reboli AC, Rotstein C, Pappas PG, Chapman SW, Kett

DH, Kumar D, et al; for the Anidulafungin Study Group. Anidulafungin versus fluconazole for invasive candidiasis. N Engl J Med 2007;356:2472–82.

[PubMed Link]

20. M.M.’s central venous catheter was changed when the positive cultures for yeast came back, and antifungal therapy per your suggestion was started on the same day (day 1). The patient receives 10 more days of par-enteral nutrition (i.e., stopped on day 11). Daily blood cultures were drawn, and all were negative starting from the ones drawn on day 4. The isolated yeast was suscep-tible to the chosen antifungal agent, and she responded appropriately to therapy. What is the most appropri-ate day to stop M.M.’s antifungal treatment? A. Day 25 (2 weeks after total parenteral nutrition

stopped).B. Day 8.C. Day 22.D. Day 14.

PSAP-VII • Critical and Urgent Care10Answers

20. Answer: CAccording to the 2009 IDSA guidelines, appropriate treat-ment duration for candidemia is 2 weeks after the first negative blood culture; although this would have been appropriate on day 18, day 22 is the earliest answer option provided that is correct (Answer C is correct). It is not appropriate to discontinue therapy after 2 weeks unless a repeat negative culture has been obtained to exclude met-astatic sites of infection (Answer D is incorrect). Unless the patient has other risk factors for invasive candidiasis, prophylactic antifungal therapy should not be continued (intravenously or orally) for the entire time the patient is in the ICU or on parenteral nutrition (Answer A is incorrect). Therapy for less than 2 weeks with documented fungemia is not recommended (Answer B is incorrect).1. Pappas PG, Kauffman CA, Andes D, Benjamin DK, Calandra

TF, Edwards JE, et al. Clinical practice guidelines for the man-agement of candidiasis: 2009 update by the Infectious Diseases Society of America. Clin Infect Dis 2009;48:503–35.

[PubMed Link]2. Ostrosky-Zeichner L, Pappas PG. Invasive candidiasis in the

intensive care unit. Crit Care Med 2006;34:857–63. [PubMed Link]

Contemporary Issues and Novel Strategies to Manage Infections in the Critically Ill

For questions 21–23 use the antibiogram.

21. You are asked to provide a recommendation to your hospital’s Pharmacy and Therapeutics Committee regarding management of the current antibiotic for-mulary. The institution is considering the removal of restrictions on several drugs because of the increase in antimicrobial resistance noted on its most recent anti-biogram. Given the most recent antibiogram data (see antibiogram), for which of the following drugs would restrictions best be removed? A. Ciprofloxacin and levofloxacin.B. Cefepime.C. Imipenem.D. Piperacillin/tazobactam.

21. Answer: DRestrictions are typically implemented to reduce misuse of an agent; reduce proper use of an expensive agent when other, less-expensive options are available; or reduce use of an agent to “save” it for situations when no other drug is effective. Many institutions use restrictions as part of an antimicrobial stewardship program to reduce overall antimicrobial drug expenditures and reduce or slow anti-biotic resistance. However, clinicians must have antibiotics

available to use empirically, and the formulary should be tailored to assist them in selecting the most appropriate agent(s). Inappropriate empiric antibiotic selection has been associated with extended length of hospital stay and increased mortality. At this hospital, ciprofloxacin, levoflox-acin, cefepime, and imipenem have reduced activity against P. aeruginosa and K. pneumoniae, with no agent retain-ing activity against more than 90% of the isolates tested. Imipenem retains excellent activity against E. cloacae, but the fluoroquinolones do not. Piperacillin/tazobactam retains activity against more than 90% of P. aeruginosa and K. pneumoniae, the most common gram-negative pathogens after Escherichia coli. Removing restrictions on ciprofloxa-cin and levofloxacin, cefepime, and imipenem would not allow choices to improve empiric antibiotic therapy for the organisms outlined above (Answer A, Answer B, and Answer C are incorrect). Directing clinicians to empiric use of piperacillin/tazobactam at this institution is the appro-priate choice to provide the greatest activity against the common gram-negative pathogens (Answer D is correct).1. Kollef MH. Broad-spectrum antimicrobials and the treatment

of serious bacterial infections: getting it right up front. Clin Infect Dis 2008;47:S3–S13.

[PubMed Link]2. Drew RH. Antimicrobial stewardship programs: how to start

and steer a successful program. J Manag Care Pharm 2009;15(2 suppl):S18–S23.

[PubMed Link]

Questions 22 and 23 pertain to the following case. Your institution is considering the addition of several new antibiotics to the formulary. However, economic conditions warrant adding only one agent at this time. Gingapenem is an intravenous carbapenem with in vitro potency against resistant gram-positive organisms including methicillin-resistant Staphylococcus aureus (MRSA) and vancomycin-resistant enterococci (VRE), as well as extended-spectrum β-lactamase (ESBL)-producing Escherichia coli, Klebsielleae pneumoniae, and Enterobacter cloacae, but no activity against Pseudomonas aeruginosa. Miraclepenem is an intravenous carbapenem with potency against a broad spectrum of gram-positive pathogens (including MRSA) and gram-negative pathogens, but not P. aeruginosa or Acinetobacter baumannii; it is not stable to ESBL or carbapenemase. Biconacillin/micobactam is an intravenous β-lactam/β-lactamase inhibitor combination with stability to class A, B, and C β-lactamases, including ESBLs. In vitro studies with this agent show it has a broad spectrum of gram-negative pathogens, including Acinetobacter spp., nonfermenting bacilli (e.g., P. aeruginosa, Stenotrophomonas maltophilia), and Enterobacteriaceae with known β-lactamases and carbapenemases. Piratecin is a novel agent with potent in vitro activity against P. aeruginosa but with less clinically effective activity against Acinetobacter spp. and ESBL-producing compounds. The cost of each agent is similar.

PSAP-VII • Critical and Urgent Care 11 Answers

Bact

eria

pat

hoge

ns is

olat

ed n

umbe

rs ar

e rep

orte

d as

per

cent

susc

eptib

le an

tibio

gram

200

8 (g

ram

-neg

ativ

e org

anism

s)

Org

anism

s G

ram

-neg

ativ

e A

ll Si

tes

Sten

otro

phom

onas

m

alto

phili

an

= 13

5

Acin

etob

acte

r ba

uman

nii

n =

109

Esch

erich

ia co

lin

= 53

83

Pseu

dom

onas

ae

rugi

nosa

n =

1193

Kleb

siella

pn

eum

onia

ea,b

n =

1306

Prot

eus

mira

bilis

n =

725

Ente

roba

cter

cloac

aen

= 42

6

Ente

roba

cter

aero

gene

sn

= 14

1

Serr

atia

m

arce

scen

sn

= 17

4A

mik

acin

9910

097

9610

097

9999

Am

oxic

illin

/clav

ulin

8080

Am

pici

llin

5573

Am

pici

llin/

sulb

acta

m82

6375

87A

ztre

onam

9761

7695

7891

99C

efaz

olin

9178

90C

efep

ime

7398

8787

9696

100

100

Cef

otax

ime

9981

Cef

otet

an99

8210

098

Ceft

riaxo

ne96

8196

7992

99C

efur

oxim

e sod

ium

8574

9924

94C

ipro

floxa

cin

7479

7082

7388

9796

Gen

tam

icin

8092

8884

9393

9899

Imip

enem

8210

088

8776

100

100

99Le

voflo

xaci

n79

7968

8176

8998

98M

erop

enem

8210

088

8776

100

100

99N

itrof

uran

toin

c95

2730

16Pi

pera

cilli

n/ta

zoba

ctam

7899

9698

9982

9199

Tetra

cycl

ine

7691

33To

bram

ycin

8394

8984

9794

9997

Trim

etho

prim

/su

lfam

etho

xazo

le84

7777

7875

9496

99a K

. pne

umon

iae c

arba

pene

mas

e det

ecte

d in

13%

of s

train

s.b Ex

tend

ed-s

pect

rum

β-la

ctam

ase i

dent

ified

in 4

6% o

f stra

ins.

c Nitr

ofur

anto

in –

use

d on

ly fo

r urin

e.

PSAP-VII • Critical and Urgent Care12Answers

22. Given the antibiogram, which agent best meets the hospital’s requirements and should be admitted to the formulary?A. Gingapenem.B. Miraclepenem.C. Biconacillin/micobactam.D. Piratecin.

22. Answer: CGingapenem will be helpful against extended-spectrum β-lactamase (ESBL)-producing E. coli, K. pneumonia, and E. cloacae, which are notable issues at this institution, but not against P. aeruginosa (Answer A is incorrect). Miraclepenem is not effective against P. aeruginosa, A. baumannii, or ESBL-producing organisms, all of which are concerns at this institution; thus, it may be a poor selection (Answer B is incorrect). Biconacillin/microbactam is the best choice given its efficacy against several resistance problems at this institution, including ESBL-producing organisms, Acinetobacter spp., P. aeruginosa, S. maltophilia, and other Enterobacteriaceae (Answer C is correct). Piratecin is active against P. aeruginosa but less so against Acinetobacter spp. and ESBL-producing compounds, making it a poor selection for this hospital (Answer D is incorrect).1. Giamarellou H, Poulakou G. Multidrug-resistant gram-neg-

ative infections: what are the treatment options? Drugs 2009;69:1879–901.

[PubMed Link]2. Zahar JR, Lortholary O, Martin C, Potel G, Plesiat P, Nordmann

P. Addressing the challenge of extended-spectrum beta-lac-tamases. Curr Opin Investig Drugs 2009;10:172–80.

[PubMed Link]

23. Which one of the following antibiotics best addresses the unmet needs described by the FDA regarding future antibiotic development strategies? A. Gingapenem.B. Miraclepenem.C. Biconacillin/micobactam.D. Piratecin.

23. Answer: CThe notable gap in the development of future antibiot-ics noted by the FDA is in antibiotics with activity against carbapenemases. Of the four compounds described, only biconacillin/microbactam has activity for these organ-isms (Answer C is correct). The other choices listed do not address this need (Answer A, Answer B, and Answer D are incorrect).1. Boucher HW, Talbot GH, Bradley JS, Edwards JE, Gilbert D,

Rice LB, et al. Bad bugs, no drugs: no ESKAPE! An update from the Infectious Diseases Society of America. Clin Infect Dis 2009;48:1–12.

[PubMed Link]

2. Theuretzbacher U. Future antibiotics scenarios: is the tide starting to turn? Int J Antimicrob Agents 2009;34:15–20.

[PubMed Link]

24. For which of the following organism and minimum inhibitory concentration (MIC) pairs is a 4-hour infusion of doripenem 1 g (administered every 8 hours) the best choice?A. E. coli when the MIC is 2 mcg/mL.B. P. aeruginosa when the MIC is 8 mg/mL.C. P. aeruginosa when the MIC is 1 mcg/mL.D. K. pneumoniae when the MIC is 2 mcg/mL.

24. Answer: BFor carbapenems, extended infusion (4-hour infusion) is an advantage over bolus administration (30–60 minutes) when the minimum inhibitory concentration (MIC) of the pathogen is high. Pseudomonas aeruginosa is difficult to treat, and it would be an appropriate use of extended infusion with the elevated MIC of 8 mcg/mL (Answer B is correct). Using Table 2-2 as a guide, the other options would all be expected to achieve target attainment probabil-ities greater than 0.9 with bolus dosing (Answer A, Answer C, and Answer D are incorrect).1. Van Wart SA, Andes DR, Ambrose PG, Bhavnania SM.

Pharmacokinetic-pharmacodynamic modeling to support doripenem dose regimen optimization for critically ill patients. Diagn Microbiol Infect Dis 2009;63:409–14.

[PubMed Link]2. Bhavnani SM, Hammel JP, Cirincione BB, Wikler MA, Ambrose

PG. Use of pharmacokinetic-pharmacodynamic target attain-ment analyses to support phase 2 and 3 dosing strategies for doripenem. Antimicrob Agents Chemother 2005;49:3944–7.

[PubMed Link]

25. You are asked to recommend empiric antibiotic therapy for an adult patient in the intensive care unit (ICU) with a diagnosis of ventilator-associated pneumonia. A bronchioloalveolar lavage sample undergoes Gram stain and shows many neutrophils, numerous gram-negative rods, and a few gram-positive cocci in clusters. In addition to vancomycin, which of the following is the best antibiotic regimen for this patient? A. Amikacin plus imipenem.B. Cefepime.C. Levofloxacin plus gentamicin.D. Piperacillin/tazobactam.

25. Answer: AThis patient requires empiric activity against the most com-mon gram-negative pathogens in the lung, including P. aeruginosa, K. pneumoniae, and Enterobacter spp., together with empiric methicillin-resistant Staphylococcus aureus

PSAP-VII • Critical and Urgent Care 13 Answers

(MRSA) coverage. Using imipenem, amikacin, and van-comycin provides empiric activity against more than 95% of the gram-negative pathogens (Answer A is correct). The presence of carbapenemase and ESBL in the institutional milieu is a strong argument against empiric coverage with cefepime (Answer B is incorrect) or piperacillin/tazo-bactam (Answer D is incorrect) alone (for gram-negative coverage). Neither levofloxacin nor gentamicin provides activity against more than 90% of the P. aeruginosa (Answer C is incorrect).1. American Thoracic Society (ATS), Infectious Diseases

Society of America (IDSA). Guidelines for the management of adults with hospital-acquired, ventilator-associated, and health care-associated pneumonia. Am J Respir Crit Care Med 2005;171:388–416.

[PubMed Link]2. Paterson DL. Resistance in gram-negative bacteria:

Enterobacteriaceae. Am J Infect Control 2006;34:S20–S28. [PubMed Link]

26. You are consulted about a patient with ventilator-associated pneumonia secondary to P. aeruginosa. The organism has the following MIC characteristics: doripenem 1 mcg/mL and piperacillin/tazobactam 8 mcg/mL. Which of the following is the best antibi-otic regimen for this patient? A. Doripenem 500 mg intravenously over 4 hours

every 8 hours.B. Doripenem 500 mg intravenously over 1 hour

every 8 hours.C. Piperacillin/tazobactam 13.5 g over 24 hours by

continuous infusion.D. Piperacillin/tazobactam 18 g over 24 hours by

continuous infusion.

26. Answer: BThis organism is inhibited by relatively low concentrations of both doripenem and piperacillin/tazobactam. Using a Monte Carlo simulation to assess the performance of various doripenem-dosing regimens in attaining pharma-cokinetic-pharmacodynamic targets for a range of MICs, one study showed that the probability of achieving 40% of the dosing interval time above an MIC of 1 mcg/mL can be achieved for doripenem using a 1-hour infusion (Answer B is correct). Doripenem 500 mg intravenously over 4 hours every 8 hours, is incorrect because the extended infusion is not required (Answer A is incorrect). A similar study showed that at an MIC of 8 mcg/mL, piperacillin/tazo-bactam 3.375 g every 6 hours administered by 30-minute intermittent infusion would provide at least 90% probability of attainment of a free serum drug concentration exceed-ing the MIC of the organism for at least 50% of the dosing interval. The continuous-infusion strategy is not required in this case (Answer C and Answer D are incorrect).

1. Bhavnani SM, Hammel JP, Cirincione BB, Wikler MA, Ambrose PG. Use of pharmacokinetic-pharmacodynamic target attain-ment analyses to support phase 2 and 3 dosing strategies for doripenem. Antimicrob Agents Chemother 2005;49:3944–7.

[PubMed Link]2. Kim A, Sutherland CA, Kuti JL, Nicolau DP. Optimal dosing

of piperacillin-tazobactam for the treatment of Pseudomonas aeruginosa infections: prolonged or continuous infusion? Pharmacotherapy 2007;27:1490–7.

[PubMed Link]

27. An ICU patient receives a diagnosis of P. aeruginosa bacteremia. Your institution uses an automated system for identification and susceptibility testing of gram-negative bacteria. The clinical microbiology laboratory reports the following susceptibility pattern for this patient’s organism:

Amikacin SusceptibleCefepime ResistantCiprofloxacin ResistantDoripenem ResistantGentamicin IntermediateImipenem ResistantPiperacillin/tazobactam Resistant

Which of the following is the best recommendation for managing this infection? A. Administer amikacin 500 mg intravenously every

6 hours.B. Perform MIC testing (using E-test) against each

of the agents listed.C. Perform MIC testing (using E-test) against

colistin.D. Administer gentamicin 7 mg/kg once daily

intravenously.

27. Answer: BMany clinical microbiology laboratories now use auto-mated susceptibility testing systems, which perform only breakpoint testing for a small range of dilutions around the susceptibility and resistance breakpoints. If an organism’s susceptibility breakpoint is 2 mcg/mL, and the resistance breakpoint is 8 mcg/mL, the systems may only test these concentrations. If the organism fails to grow at 2 mcg/mL, the report of “susceptible” is issued, although the true MIC of the organism could be much lower (e.g., 0.125 mcg/mL). If the organism grows at both concentrations, then “resistant” is reported, although the MIC could be just one dilution higher (e.g., 16 mcg/mL) and be treated with higher than standard dosing of the drug. For this organism, performing MIC testing using the E-test will provide exact MIC data, allowing a tailoring of drug selection and dos-ing (Answer B is correct). Using an aminoglycoside alone is not recommended (Answer A and Answer D are incorrect).

PSAP-VII • Critical and Urgent Care14Answers

Although colistin is an option, it should not be used until the full extent of the MIC data for the other agents are known (Answer C is incorrect).1. Kuti JL, Shore E, Palter M, Nicolau DP. Tackling empirical

antibiotic therapy for ventilator-associated pneumonia in your ICU: guidance for implementing the guidelines. Semin Respir Crit Care Med 2009;30:102–15.

[PubMed Link]2. Nicasio AM, Eagye KJ, Nicolau DP, Shore E, Palter M, Pepe J,

Kuti JL. Pharmacodynamic-based clinical pathway for empiric antibiotic choice in patients with ventilator-associated pneu-monia. J Crit Care 2010;25:69–77.

[PubMed Link]

28. You are consulted about a patient with a K. pneumoniae bacteremia. The organism has the following MIC char-acteristics: levofloxacin 8 mcg/mL, doripenem 8 mcg/ mL, and piperacillin/tazobactam 64 mcg/mL. Which antibiotic regimen is best for this patient?A. Doripenem 1 g intravenously over 4 hours every 8

hours.B. Levofloxacin 750 mg intravenously every 24

hours.C. Piperacillin/tazobactam 13.5 g over 24 hours by

continuous infusion.D. Piperacillin/tazobactam 18 g over 24 hours by

continuous infusion.

28. Answer: AThis organism has relatively higher MICs for all antibiotics tested. An extended infusion of doripenem 1 g over 4 hours has a 99% probability of achieving the target pharmaco-dynamic parameter and is the most appropriate treatment regimen (Answer A is correct). The organism should be interpreted as resistant to levofloxacin (Answer B is incor-rect) and piperacillin/tazobactam (Answer C and Answer D are incorrect).1. Bhavnani SM, Hammel JP, Cirincione BB, Wikler MA, Ambrose

PG. Use of pharmacokinetic-pharmacodynamic target attain-ment analyses to support phase 2 and 3 dosing strategies for doripenem. Antimicrob Agents Chemother 2005;49:3944–7.

[PubMed Link]2. Nicasio AM, Eagye KJ, Nicolau DP, Shore E, Palter M, Pepe J,

Kuti JL. Pharmacodynamic-based clinical pathway for empiric antibiotic choice in patients with ventilator-associated pneu-monia. J Crit Care 2010;25:69–77.

[PubMed Link]

29. Which of the following antibiotic regimens for P. aeruginosa is most likely to generate resistance dur-ing therapy? A. Doripenem (MIC 2 mcg/mL) 500 mg

intravenously over 4 hours every 8 hours.

B. Piperacillin/tazobactam (MIC 8 mcg/mL) 4.5 g intravenously over 30 minutes every 6 hours.

C. Ciprofloxacin (MIC 4 mcg/mL) 400 mg intravenously over 30 minutes every 12 hours.

D. Cefepime (MIC 4 mcg/mL) 2 g intravenously over 4 hours every 6 hours.

29. Answer: CAntibiotic underdosing correlates with resistance forma-tion, particularly with fluoroquinolones. Ciprofloxacin 400 mg intravenously every 12 hours is unlikely to be effective unless the MIC is less than 0.3 mcg/mL, and it will most likely promote resistance generation while on therapy (Answer C is correct). Doripenem, piperacillin/tazobac-tam, and cefepime at the doses and regimens given have more than a 90% probability of reaching their target phar-macodynamic parameter and eradicating the organism (Answer A, Answer B, and Answer D are incorrect).1. Lee SY, Kuti JL, Nicolau DL. Cefepime pharmacodynamics in

patients with extended spectrum b-lactamase (ESBL) and non-ESBL infections. J Infect 2007;54:463–8.

[PubMed Link]2. Kim A, Sutherland CA, Kuti JL, Nicolau DP. Optimal dosing

of piperacillin-tazobactam for the treatment of Pseudomonas aeruginosa infections: prolonged or continuous infusion? Pharmacotherapy 2007;27:1490–7.

[PubMed Link]3. Bhavnani SM, Hammel JP, Cirincione BB, Wikler MA, Ambrose

PG. Use of pharmacokinetic-pharmacodynamic target attain-ment analyses to support phase 2 and 3 dosing strategies for doripenem. Antimicrob Agents Chemother 2005;49:3944–7.

[PubMed Link]

30. Your pharmacy department has recommended 12 smart pumps for the ICU. What is the best justifica-tion for this pharmacy department purchase? A. The pumps will facilitate extended or continuous

infusions of antibiotics, reducing drug expenditures and mitigating resistance issues.

B. The pumps will reduce nursing time for drug administration, freeing personnel for other patient care activities.

C. Alerts and stops provided by the pumps will reduce programming errors and miscalculated doses that might have resulted in patient harm.

D. The pumps can be integrated with the pharmacy computer system to alert the pharmacy when a dose has been missed.

30. Answer: AExtended and continuous infusion of β-lactams is not in mainstream use in U.S. hospitals, largely because of the challenge in successfully administering the drugs for the correct duration. However, this strategy is best for saving

PSAP-VII • Critical and Urgent Care 15 Answers

the pharmacy department drug acquisition costs and will potentially reduce antibiotic resistance (Answer A is cor-rect). Although nursing time may be reduced for the administration of extended or continuous infusions, this strategy does not affect the pharmacy department budget (Answer B is incorrect). Patient safety is improved with smart pump integration, and it may be possible to integrate the smart pump data into the pharmacy computer system to identify missed doses; however, these advantages are unlikely to justify the purchase of 12 pumps by the phar-macy department (Answer C and Answer D are incorrect).1. Trbovich PL, Pinkney S, Cafazzo JA, Easty AC. The impact of traditional and smart pump infusion technology on nurse medication administration performance in a simulated inpatient unit. Qual Saf Health Care 2010 Apr 27 [Epub ahead of print]. [PubMed Link]2. Rothschild JM, Keohane CA, Cook EF, Orav EJ, Burdick E, Thompson S, et al. A controlled trial of smart infusion pumps to improve medication safety in critically ill patients. Crit Care Med 2005;33:533–40. [PubMed Link]

31. On rounds in the ICU, you observe a patient with a positive blood culture (one of two drawn) for gram-positive cocci in clusters. The patient is clinically stable and is not taking antibiotics. What is the best course of action for this patient? A. Request a peptide nucleic acid fluorescent in situ

hybridization (PNA FISH) S. aureus assay to rule out contaminated blood culture with coagulase-negative staphylococci (CNS).

B. Request a BD GeneOhm blood culture assay to rule out MRSA.

C. Initiate vancomycin empirically and await species identification.

D. Initiate telavancin empirically and await species identification.

31. Answer: AThe peptide nucleic acid fluorescence in situ hybridiza-tion (PNA FISH) S. aureus assay will differentiate between S. aureus and coagulase-negative staphylococci ( CNS) (Answer A is correct). This allows discontinuation of anti-biotics in patients with contaminated blood cultures. The BD GeneOhm does not differentiate CNS from S. aureus (Answer B is incorrect). The other options may lead to unnecessary use of antimicrobials (Answer C and Answer D are incorrect).1. Procop GW. Molecular diagnostics for the detection and

characterization of microbial pathogens. Clin Infect Dis 2007;45(suppl 2):S99–S111.

[PubMed Link]2. Weile J, Knabbe C. Current applications and future

trends of molecular diagnostics in clinical bacteriology. Anal Bioanal Chem 2009;394:731–42.

[PubMed Link]

32. A large urban community hospital implements pro-tocol-driven projects for community-acquired pneumonia and sepsis. During the first 12 months, use of the protocols was 10% to 15% for all qualified patients. A new clinical decision support (CDS) system is purchased to increase compliance with evidence-based protocols. One of the decentralized pharmacists at this hospital is hesitant to make recommendations based on the new CDS system because they may inter-fere with her better clinical judgment. Which of the following educational statements is best to increase her compliance with protocol use? A. Several clinical studies have reported improved

outcomes associated with CDS systems.B. CDS systems have been designed with

the flexibility for clinicians to override recommendations when clinical judgment conflicts.

C. Protocol compliance was improved by more than 50% in similar-sized institutions when a CDS system was implemented.

D. CDS systems can be customized to follow nationally published guidelines.

32. Answer: BClinical decision support systems do not replace clinical judgment—sometimes, the recommendation of the sys-tem will be overridden (Answer B is correct). Providing literature support and evidence from other institutions is appropriate but may not allay the fears of the clinical deci-sion support (CDS) system, leading to liability (Answer A and Answer C are incorrect). Using nationally published guidelines as a basis for recommendations is appropriate, but guidelines are not available for all diseases and thera-peutic situations (Answer D is incorrect).1. Sintchenko V, Coiera E, Iredell JR, Gilbert GL. Comparative

impact of guidelines, clinical data and decision support on prescribing decisions: an interactive Web experiment with sim-ulated cases. J Am Med Inform Assoc 2004;11:71–7.

[PubMed Link]2. Buising KL, Thursky KA, Black JF, MacGregor L, Street AC,

Kennedy MP, et al. Improving antibiotic prescribing for adults with community-acquired pneumonia: does a computerised decision support system achieve more than academic detail-ing alone? A time series analysis. BMC Med Inform Decis Mak 2008;8:35.

[PubMed Link]

33. An important concern at your institution among pharmacy management is that clinicians follow only nationally recognized treatment guidelines for com-munity-acquired pneumonia in 25% of patients with the diagnosis. Which of the following decision sup-

PSAP-VII • Critical and Urgent Care16Answers

port systems will best improve the percentage of patients receiving guideline-based therapy? A. Passive system with real-time antibiogram data

and microbiology reports.B. Active system with real-time antibiogram data

and microbiology reports.C. Passive system with antibiotic guidelines.D. Active system with antibiotic guidelines.

33. Answer: AIt is important to allow clinicians to preserve their clinical decision-making even on basic, straightforward situations so that acceptance of the CDS is gained; this is accom-plished with a passive decision support system (Answer A is correct). An active system does not allow clinician par-ticipation in decision-making (Answer B and Answer D are incorrect). A passive system with antibiotic guidelines does not incorporate tracking of antimicrobial use and real-time generation of antibiograms to allow the clinician the ability to assess resistance patterns within a certain unit or hospi-tal in a system at any given time as opposed to waiting for a yearly report (Answer C is incorrect).1. Sintchenko V, Coiera E, Iredell JR, Gilbert GL. Comparative

impact of guidelines, clinical data and decision support on prescribing decisions: an interactive Web experiment with sim-ulated cases. J Am Med Inform Assoc 2004;11:71–7.

[PubMed Link]2. Deuster S, Roten I, Muehlebach S. Implementation of treat-

ment guidelines to support judicious use of antibiotic therapy. J Clin Pharm Ther 2010;35:71–8.

[PubMed Link]

34. To decrease time to notification of MRSA-colonized ICU patients, your institution is considering the pur-chase of a rapid diagnostic system. The microbiology laboratory director would also like to use the system for diagnosing MRSA from suspected infections. With limited resources available, only one system can be pur-chased. Which one of the following would provide the broadest function for MRSA identification? A. PNA FISH S. aureus assay.B. GeneOhm MRSA assay.C. Gene Xpert MRSA assay.D. Xpert MRSA/SA skin and skin structure assay.

34. Answer: CNo single system will both detect colonization and dif-ferentiate MRSA from methicillin-sensitive S. aureus (MSSA) in active infection. The Gene Xpert MRSA assay would be the best starting point for the laboratory to begin detecting MRSA colonization (Answer C is correct). The Xpert MRSA/SA skin and skin structure assay could be added later because the two operate on the same platform (Answer D is incorrect). The PNA FISH does not detect

colonization (Answer A is incorrect). The BD GeneOhm assay does detect colonization but would not allow the detection of MRSA from suspected sites of infection (Answer B is incorrect).1. Procop GW. Molecular diagnostics for the detection and

characterization of microbial pathogens. Clin Infect Dis 2007;45(suppl 2):S99–S111.

[PubMed Link]2. Weile J, Knabbe C. Current applications and future

trends of molecular diagnostics in clinical bacteriology. Anal Bioanal Chem 2009;394:731–42.

[PubMed Link]

35. Six months ago, your hospital implemented a CDS system. One infectious diseases physician refuses to review the alerts from the system, stating that he does not need any help. Which of the following is the best option for improving this physician’s compliance with the decision support recommendations?A. Present data on cost-savings from the literature

and from other institutions to the physician.B. Ask him to test the system by following its

recommendations for 1 month and then reevaluate.

C. Organize a site visit to a nearby medical center that has implemented the system under consideration.

D. Remind him that his colleagues are using the program and finding it helpful.

35. Answer: BBy inviting the physician to “test” the system, the threat of lost autonomy and control is removed, and the physician is able to assess the software using his clinical skills (Answer B is correct). Making cost savings the primary reason for implementation may alienate the physician, who rightly believes he is providing the best care for his patients, no matter the cost (Answer A is incorrect). Bullying the phy-sician through peer pressure may have the opposite effect, cementing his opposition to compliance with a software-driven decision model (Answer C and Answer D are incorrect).1. Sintchenko V, Coiera E, Iredell JR, Gilbert GL. Comparative

impact of guidelines, clinical data and decision support on prescribing decisions: an interactive Web experiment with sim-ulated cases. J Am Med Inform Assoc 2004;11:71–7.

[PubMed Link]2. Thursky K. Use of computerized decision support systems to

improve antibiotic prescribing. Expert Rev Anti Infect Ther 2006;4:491–507.

[PubMed Link]

PSAP-VII • Critical and Urgent Care 17 Answers

36. A PNA FISH S. aureus assay was implemented 3 months ago at your institution. You have noticed that, many times, the resulting change in antibiotic therapy does not occur until the morning after the completion and reporting of the test results. Which of the follow-ing is the best next step?A. Collaborate with the laboratory to notify

prescribers immediately of the results.B. Educate the nursing staff on the importance of

timely action with the test results.C. Monitor for 3 more months to see whether the

timing improves.D. Educate the physicians on the importance of

timely action with the test results.

36. Answer: AEffective communication of results from laboratory to pre-scriber in a timely fashion is the most likely way to improve the rapidity of therapy adjustment (Answer A is correct). The benefits of rapid diagnostics are lost if the information is placed in an electronic or paper record and not noted by the prescriber until hours or days later. The system must include a method (e.g., fax, telephone call, text alert) to notify the prescriber when the test results are available; optimally, this would occur before rounds. Nursing education is impor-tant, but communication with the prescriber is the barrier to therapy change, not nurses’ ignorance of the importance of the system (Answer B is incorrect). Additional study without intervention is unlikely to improve time to imple-mentation of therapeutic changes (Answer C is incorrect). Physician ignorance of the benefits of the testing system does not appear to be the problem (Answer D is incorrect); having the proper information faster will likely improve patient care.1. Hermsen ED, Shull SS, Klepser DG, Iwen PC, Armbrust A,

Garrett J, et al. Pharmacoeconomic analysis of microbiologic techniques for differentiating staphylococci directly from blood culture bottles. J Clin Microbiol 2008;46:2924–9.

[PubMed Link]2. Struelens MJ, Hawkey PM, French GL, Witte W, Tacconelli

E. Laboratory tools and strategies for methicillin-resistant Staphylococcus aureus screening, surveillance and typ-ing: state of the art and unmet needs. Clin Microbiol Infect 2009;15:112–9.

[PubMed Link]

37. You wish to convince the hospital administration of the importance of instituting a new rapid molecular diagnostic that will differentiate MRSA versus methi-cillin-sensitive S. aureus (MSSA) from blood cultures. Which of the following arguments is best to present to your administrators? A. Antibiotic use will be considerably decreased.

B. Patients initiated on daptomycin will be changed to cefazolin sooner.

C. Patients initiated on ceftriaxone empirically will be changed to vancomycin sooner.

D. Expenses for isolation procedures will decrease.

37. Answer: CThe most important clinical benefit of rapid diagnostics is making sure that appropriate therapy is initiated as rapidly as possible. Identification of an MRSA not currently cov-ered, such as changing ceftriaxone to vancomycin, allows appropriate initial therapy to prevent morbidity and mor-tality (Answer C is correct). Deescalating from daptomycin to cefazolin in a patient with MSSA is also an important benefit of rapid diagnostics, but it is less critical to patient outcome (Answer B is incorrect). Rapid diagnostics may lead to faster appropriate therapy and improved outcomes, but no clear data exist to show decreased antibiotic use attributable to the testing (Answer A is incorrect). Isolation expenses will not be directly related to the results of the PNA FISH assay (Answer D is incorrect).1. Cunningham R, Jenks P, Northwood J, Wallis M, Ferguson S,

Hunt S. Effect on MRSA transmission of rapid PCR testing of patients admitted to critical care. J Hosp Infect 2007;65:24–8.

[PubMed Link]2. Harbarth S, Masuet-Aumatell C, Schrenzel J, Francois P,

Akakpo C, Renzi G, et al. Evaluation of rapid screening and pre-emptive contact isolation for detecting and controlling methicillin-resistant Staphylococcus aureus in critical care: an interventional cohort study. Crit Care 2006;10:R25.

[PubMed Link]

38. Laboratory, pharmacy, radiology, and nursing docu-mentation systems in your hospital are each running separate software systems that do not interface well. As a result, compliance with bacteremia, community-acquired pneumonia, and sepsis bundles is poor. Your health care system operated at a deficit in the past fiscal year. What is best to recommend that administra-tors consider as they choose a CDS system for your hospital? A. Support from infectious diseases physicians.B. Compatibility with software from ancillary

departments.C. Price of the system.D. Support from the vendor when new evidence-

based literature becomes available.

38. Answer: BIntegration of information systems is the most likely cause for poor compliance with infectious disease treatment bundles. Improving compatibility of software from the various information-reporting departments holds the best promise to successful implementation of a CDS (Answer

PSAP-VII • Critical and Urgent Care18Answers

B is correct). Infectious diseases physicians would not be expected to impose barriers to implementation of a CDS (Answer A is incorrect). Although the price of the system is important, investment in a less-expensive system that does not integrate many software systems would be fool-ish (Answer C is incorrect). Updates are necessary, but the current problems with bundle compliance are not likely because of outdated bundles, but rather poor sharing of information (Answer D is incorrect).1. Pestotnik MS. Expert clinical decision support sys-

tems to enhance antimicrobial stewardship programs. Pharmacotherapy 2005:25:1116–25.

[PubMed Link]2. Steurbaut K, Van Hoecke S, Colpaert K, Lamont K, Taveirne K,

Depuydt P, et al. Use of web services for computerized medi-cal decision support, including infection control and antibiotic management, in the intensive care unit. J Telemed Telecare 2010;16:25-9.

[PubMed Link]

39. Your institution is considering real-time polymerase chain reaction for the detection of Clostridium diffi-cile in ICU patients. Which of the following is most likely to result from this change? A. Decreased use of oral vancomycin.B. Decreased turnaround time from the

microbiology laboratory.C. Increased detection of C. difficile cases.D. Improvement in the ease of C. difficile diagnosis.

39. Answer: CBecause polymerase chain reaction (PCR) testing is more sensitive than currently used C. difficile assays in the micro-biology laboratory, increased detection of C. difficile cases is the probable outcome (Answer C is correct). Oral vanco-mycin use will not necessarily be decreased; it may possibly increase if more cases are detected (Answer A is incorrect). The turnaround time for the C. difficile PCR is similar to that for the enzyme immunoassay currently used in most laboratories (Answer B is incorrect). C. difficile diagnosis will still depend on correlation with the clinical condition of the patient (Answer D is incorrect).1. Conterno LO, Shymanski J, Ramotar K, Toye B, van Walraven

C, Coyle D, et al. Real-time polymerase chain reaction detec-tion of methicillin-resistant Staphylococcus aureus: impact on nosocomial transmission and costs. Infect Control Hosp Epidemiol 2007;28:1134–41.

[PubMed Link]2. Goldenberg SD, Dieringer T, French GL. Detection of toxi-

genic Clostridium difficile in diarrheal stools by rapid real-time polymerase chain reaction. Diagn Microbiol Infect Dis 2010;67:304–7.

[PubMed Link]

40. An ICU patient with a history of recent total knee replacement required ICU admission because of nar-cotic over-administration on the orthopedic ward. Last night, he developed a temperature of 40°C, with chills and delirium. Today, the microbiology laboratory has reported that the patient has two of two positive blood cultures with gram-positive cocci in clusters. Which of the following interventions is best for this patient? A. Run a PNA FISH probe to determine the

coagulase status of the organism.B. Run a GeneXpert probe to determine the

presence of methicillin resistance.C. Initiate nafcillin.D. Initiate vancomycin.

40. Answer: BRunning a GeneXpert probe to distinguish MRSA from MSSA will allow selection of appropriate empiric antimi-crobial therapy (Answer B is correct). The PNA FISH assay does not detect susceptibility (Answer A is incorrect), and because the prevalence of MRSA at most hospitals is greater than 50%, methicillin resistance should be anticipated until susceptibilities are known. Because the patient has two positive blood cultures, this is unlikely to be because of contamination. Initiating nafcillin would not be appropri-ate until methicillin resistance is disproved (Answer C is incorrect). Starting vancomycin would be appropriate but may not be necessary based on results of above rapid diag-nostics (Answer D is incorrect).1. Cunningham R, Jenks P, Northwood J, Wallis M, Ferguson S,

Hunt S. Effect on MRSA transmission of rapid PCR testing of patients admitted to critical care. J Hosp Infect 2007;65:24–8.

[PubMed Link]2. Carroll K. Rapid diagnostics for methicillin-resistant

Staphylococcus aureus. Mol Diagn Ther 2008;12:15–24. [PubMed Link]

Update in Severe Sepsis and Septic Shock Management

Questions 41 and 42 pertain to the following case. R.S. is a 64-year-old man admitted to the medical intensive care unit (ICU) from the emergency department (ED) with presumed aspiration pneumonia. He has a history of non– small cell lung cancer (off treatment for 2 years), chronic kidney disease stage 3, and gastrointestinal bleeding 5 weeks ago secondary to chronic nonsteroidal anti-inflammatory drug use requiring endoscopic hemostasis (now on proton pump inhibitor therapy). In the ED, R.S.’s presenting blood pressure was 82/54 mm Hg, pulse rate 118 beats/minute, temperature 101.1°F (38.4°C), white blood cell (WBC) count 13.8 × 103 cells/mm3, platelet count 116,000/mm3, and hematocrit 39%. Blood cultures were

PSAP-VII • Critical and Urgent Care 19 Answers

obtained, and antibiotics were initiated within 30 minutes of presentation. Fluid resuscitation was initiated with 1 L of intravenous normal saline. Because his condition continued to acutely decline, he required emergency intubation and a norepinephrine infusion to maintain systolic blood pressure (SBP) above 90 mm Hg. R.S. was transferred to the medical ICU, where his SBP ranged from 118 mm Hg to 124 mm Hg on norepinephrine (10 mcg/minute). Pantoprazole 40 mg intravenously two times/day and subcutaneous heparin 5000 units three times/day were begun. Early goal-directed therapy, continued according to the institution’s severe sepsis resuscitation and management bundles, includes orders for drotrecogin alfa (activated) (DAA).

41. Which of the following is the most important rel-ative contraindication or warning about DAA to consider for R.S.? A. Concurrent prophylactic subcutaneous heparin.B. History of non–small cell lung cancer.C. History of gastrointestinal bleeding.D. Severity of illness (i.e., R.S. has septic shock).

41. Answer: CHistory of gastrointestinal bleeding is the most impor-tant consideration because recent (within 6 weeks) gastrointestinal bleeding is a warning against drotreco-gin alfa (activated) (DAA) administration (Answer C is correct). Drotrecogin alfa (activated) with concur-rent therapeutically dosed heparin is a warning; however, subcutaneous heparin for deep venous thrombosis prophy-laxis is not a warning or contraindication for concomitant DAA (Answer A is incorrect). Based on the results from the Xigris and Prophylactic HepaRin Evaluation in Severe Sepsis (XPRESS) study, the subcutaneous heparin should be continued. A history of non–small cell lung cancer is not an exclusion for DAA (Answer B is incorrect). Answer D is incorrect because DAA is indicated for both severe sep-sis and septic shock as defined by an Acute Physiology and Chronic Health Evaluation (APACHE II) score of 25 or more or two or more organ failures. 1. Bernard GR, Vincent JL, Laterre PF, LaRosa SP, Dhainaut JF,

Lopez-Rodriguez A, et al. Efficacy and safety of recombinant human activated protein C for severe sepsis. N Engl J Med 2001;344:699–709.

[PubMed Link]2. Levi M, Levy M, Williams MD, Douglas I, Artigas A, Antonelli

M, et al. Xigris and Prophylactic HepaRin Evaluation in Severe Sepsis (XPRESS) Study Group. Prophylactic heparin in patients with severe sepsis treated with drotrecogin alfa (acti-vated). Am J Respir Crit Care Med 2007;176:483–90.

[PubMed Link]

42. R.S. is continued on early goal-directed therapy and receives boluses of crystalloid for a 3-hour period until he achieves a central venous pressure (CVP) of 12 mm

Hg. Because of the fluid resuscitation, his norepineph-rine requirements decrease (from 20 mcg/minute to 3 mcg/minute). The physician would like to initiate hydrocortisone 50 mg intravenously every 6 hours. Which of the following is the best rationale against the use of hydrocortisone in R.S.? A. SBP is volume-responsive, and the

norepinephrine requirement is decreasing.B. Risk of hyperglycemia associated with steroids

outweighs the potential benefit.C. The state of his adrenal function is unknown and

should be evaluated first.D. The risk of infection associated with steroids

outweighs the potential benefit.

42. Answer: AR.S. has remained hemodynamically stable on norepineph-rine with an appropriate fluid challenge (central venous pressure [CVP] = 12 mm Hg), and his vasopressor require-ment subsided within 3 hours of onset; therefore, the best reason for not initiating hydrocortisone is that his systolic blood pressure (SBP) has remained stable on norepineph-rine (Answer A is correct). Hyperglycemia, although a common adverse effect of corticosteroid administration, has not been consistently shown to be worsened by corti-costeroid administration (Answer B is incorrect). Based on the results of the Corticosteroid Therapy of Septic Shock (CORTICUS) study, adrenocorticotropin (ACTH) stim-ulation tests are no longer recommended to identify the subset of patients with critical illness–related corticoste-roid deficiency (Answer C is incorrect). If corticosteroids are required for short-term treatment of volume and vaso-pressor refractory septic shock, then the benefit would outweigh the risk of infection (Answer D is incorrect).1. Dellinger RP, Levy MM, Carlet JM, Bion J, Parker MM,

Jaeschke R, et al. Surviving Sepsis Campaign: international guidelines for the management of severe sepsis and septic shock: 2008. Crit Care Med 2008;36:296–327.

[PubMed Link]2. Sprung CL, Annane D, Keh D, Moreno R, Singer M, Freivogel

K, et al. Hydrocortisone therapy for patients with septic shock. N Engl J Med 2008;358:111–24.

[PubMed Link]

43. E.F. is a 25-year-old woman with poorly controlled type 2 diabetes mellitus (glycated hemoglobin on admission of 9.2%) who was admitted to the ICU with presumed necrotizing fasciitis of the right lower extremity and resultant septic shock. Timely broad-spectrum antimicrobial therapy was initiated, and E.F. met all early goal-directed therapy end points (CVP = 14 mm Hg, central venous oxygen saturation [ScvO2] = 85%) with the administration of intrave-nous fluids, norepinephrine, and dobutamine. She also

PSAP-VII • Critical and Urgent Care20Answers

received hydrocortisone 200 mg/day for her volume and pressor-refractory septic shock. Today, on day 3 of hospitalization, E.F. has been extubated and weaned off vasopressors completely with a stable SBP greater than 140 mm Hg. She is ready to transfer from the ICU. Which one of the following is the best recommen-dation for E.F. regarding hydrocortisone? A. Complete 7 days of hydrocortisone and then stop.B. Change hydrocortisone to an equivalent dose of

oral prednisone for 7 days of therapy.C. Discontinue hydrocortisone using a short-course

tapering schedule.D. Discontinue hydrocortisone now because she is

off vasopressors.

43. Answer: CGiven that this patient is now hemodynamically stable after 3 days of hydrocortisone therapy and completely off vasopressors, the correct answer based on results of the CORTICUS study is to discontinue steroids using a short-course tapering schedule (Answer C is correct). There is no need to change hydrocortisone to an equivalent dose of oral prednisone, and 7 days of therapy are not neces-sary (Answer B is incorrect). Before CORTICUS, the usual practice would have been to complete a 7-day course of hydrocortisone; however, this recommendation ceased based on the 2008 Surviving Sepsis Campaign guidelines (Answer A is incorrect). Although it seems reasonable to discontinue the hydrocortisone given that the patient is off vasopressors, and many clinicians do advocate for this approach, it has never been studied in clinical trials and is not the optimal approach (Answer D is incorrect).1. Annane D, Sebille V, Charpentier C, Bollaert PE, Francois B,

Korach JM, et al. Effect of treatment with low doses of hydro-cortisone and fludrocortisone on mortality in patients with septic shock. JAMA 2002;288:862–71.

[PubMed Link]2. Sprung CL, Annane D, Keh D, Moreno R, Singer M, Freivogel

K, et al. Hydrocortisone therapy for patients with septic shock. N Engl J Med 2008;358:111–24.

[PubMed Link]

44. M.M. is a 76-year-old man with a history of chronic obstructive pulmonary disease (COPD) and diabe-tes. He is admitted to your hospital with severe sepsis and hypoxic respiratory failure leading to urgent air-way intubation on arrival. M.M. was transferred from a small community hospital ED, where he was pre-sumed to have COPD exacerbation and was given empiric levofloxacin (dose and route of administra-tion unknown) and nebulized albuterol every 4 hours. M.M.’s home drugs include albuterol/ipratropium 2 puffs four times/day, oxygen at night, prednisone 40 mg by mouth as needed for acute COPD exacerbations

(he has taken one or two doses in the previous week), and metformin 1000 mg/day. Vital signs on admis-sion include blood pressure 100/68 mm Hg (after 5 L of crystalloid therapy), pulse rate 88 beats/minute, temperature 100.6°F (38.1°C), WBC count 13.0 × 103

cells/mm3. Which of the following is the best course of action regarding M.M.’s steroid therapy? A. Perform an adrenocorticotropin hormone

stimulation test.B. Initiate stress dose hydrocortisone now.C. Reinitiate his home prednisone regimen.D. No steroid therapy is required.

44. Answer: DThe patient has received adequate fluid resuscitation (5 L) and is currently normotensive. In addition, a history of absolute adrenal insufficiency at baseline would support the initiation of stress dose steroids without measuring an ACTH stimulation test. He does not have a history of abso-lute adrenal insufficiency and was not on mineral-corticoid replacement on admission (Answer D is correct). Therefore, steroid therapy for septic shock is no longer applicable to this patient (Answer B is incorrect). Performing an ACTH stimulation test to determine eligibility of steroid therapy is no longer recommended, and his previous doses of predni-sone may interfere with the validity of this test (Answer A is incorrect). Although the patient may still require pred-nisone for his chronic obstructive pulmonary disease (COPD) exacerbation, this was a week ago and may no lon-ger be applicable; therefore, withholding the resumption of prednisone until he exhibits COPD symptoms is not unreasonable given that he will be monitored while in hos-pital (Answer C is incorrect). 1. Marik PE, Pastores SM, Annane D, Meduri GU, Sprung CL,

Arlt W, et al. Recommendations for the diagnosis and man-agement of corticosteroid insufficiency in critically ill adult patients: consensus statements from an international task force by the American College of Critical Care Medicine. Crit Care Med 2008;36:1937–49.

[PubMed Link]2. Annane D, Bellissant E, Bollaert PE, Briegel J, Canfalonieri M,

De Gaudio R, et al. Corticosteroids in the treatment of severe sepsis and septic shock in adults: a systematic review. JAMA 2009;301:2362–75.

[PubMed Link]

Questions 45–47 pertain to the following case. J.L., a 39-year-old man with no significant medical history, presents to the ED with suspected community-acquired pneumonia. His symptoms began 2 days ago with shortness of breath, purulent sputum production, and fever. On physical examination, J.L. weighs 85 kg, is alert and oriented, and has the following vital signs: temperature 102.7°F (39.3°C), pulse rate 102 beats/minute, respiratory rate 26 breaths/minute, and blood pressure of 74/40 mm

PSAP-VII • Critical and Urgent Care 21 Answers

Hg. Chest radiography shows left lower lobe consolidation consistent with pneumonia. Blood and respiratory samples are collected. His initial WBC count was 21.3 × 103 cells/mm3 and lactate 4.8 mmol/L. The ED physician places a central venous catheter and initiates aggressive fluid resuscitation with normal saline. After 2 L of normal saline boluses, J.L.’s mean arterial pressure (MAP) remains below 60, and he has had 30 mL of urine output in 2 hours.

45. Which of the following additional data is most helpful to guide resuscitation for J.L.?A. Pulse rate.B. CVP.C. Hematocrit.D. ScvO2.

45. Answer: BA stepwise approach in the management of severe sepsis and septic shock aids in determining which information is needed for the next step. Given that central line placement is under way, the next decision is whether additional fluid administration is needed as determined by the CVP (Answer B is correct). This data point needs evaluation before hema-tocrit and Scvo2 can be interpreted for the hemodynamic resuscitation of the patient (Answer C and Answer D are incorrect). Pulse rate is not part of the standardized resus-citation end points and is commonly confounded by other clinical factors (Answer A is incorrect).1. Zanotti Cavazzoni SL, Dellinger RP. Hemodynamic opti-

mization of sepsis-induced tissue hypoperfusion. Crit Care 2006;10(suppl 3):S2.

[PubMed Link]2. Rivers E, Nguyen B, Havstad S, Ressler J, Muzzin A, Knoblich

B, et al. Early goal-directed therapy in the treatment of severe sepsis and septic shock. N Engl J Med 2001;345:1368–77.

[PubMed Link]

46. J.L.’s initially responds to several fluid challenges; how-ever, the effect is transient. He now has a MAP of 40 mm Hg and has become increasingly confused. His hematocrit is 30%, and his serum lactate is 5.0. Which of the following options is best for J.L.? A. Continue fluids and initiate an epinephrine

infusion until urine output increases to 40 mL/ hour and he attains a MAP of 65 mm Hg or more.

B. Transfuse packed red blood cells until the CVP is 8 mm Hg or more.

C. Continue fluids and initiate a norepinephrine infusion to attain a MAP of 65 mm Hg or more.

D. Continue fluids and initiate hydrocortisone 50 mg intravenously every 6 hours.

46. Answer: C

Despite the patient’s appropriate response to fluid challenges during the next 2 hours, the effect on mean arterial pressure (MAP) is transient, and he develops life-threatening hypo-tension with apparent lack of perfusion to the central nervous system. For life-threatening hypotension, regardless of the amount of fluid administered or CVP goals, the initiation of norepinephrine to maintain a MAP of 65 mm Hg or more is the most appropriate (Answer C is correct). Epinephrine is a reasonable vasopressor choice; however, both of the recently published randomized controlled trials evaluating the role of epinephrine in severe sepsis and septic shock showed higher incidence of early adverse events, thus making it less ideal than norepinephrine, especially because his lactate is already elevated (Answer A is incorrect). Transfusion of packed red blood cells is incorrect because the initiation of this interven-tion depends on Scvo2 (Answer B is incorrect). Initiation of hydrocortisone is incorrect because this should only occur after fluid and vasopressor therapy have failed (Answer D is incorrect).1. Annane D, Vignon P, Renault A, Bollaert PE, Charpentier C,

Martin C, et al. Norepinephrine plus dobutamine versus epi-nephrine alone for management of septic shock: a randomised trial. Lancet 2007;370:676–84.

[PubMed Link]2. LeDoux D, Astiz ME, Carpati CM, Rackow EC.

Effects of perfusion pressure on tissue perfusion in septic shock. Crit Care Med 2000;28:2729–32.

[PubMed Link]3. Myburgh JA, Higgins, A, Jovanovska A, Lipman J,

Ramakrishnan N, Santamaria J. A comparison of epinephrine and norepinephrine in critically ill patients. Intensive Care Med 2008;34:2226–34.

[PubMed Link]

47. J.L. had a respiratory infection 1 month ago for which he was treated with a 10-day course of levofloxacin. In addition to gentamicin, which of the following is best to add to his antibiotic regimen?A. Ciprofloxacin.B. Cefepime.C. Ceftriaxone and vancomycin.D. Cefepime and vancomycin.

47. Answer: DThe finding of recent antibiotic therapy (10-day course of levofloxacin 1 month ago) puts the patient at risk of antibi-otic-resistant pathogens including Pseudomonas aeruginosa and methicillin-resistant Staphylococcus aureus (MRSA), as well as quinolone-resistant organisms. Ciprofloxacin is not appropriate given the need to cover for P. aeruginosa and the risk of quinolone resistance (Answer A is incorrect). This regimen also lacks activity against MRSA. Empiric cov-erage for this patient should include an antipseudomonal β-lactam plus an aminoglycoside plus an antimicrobial

PSAP-VII • Critical and Urgent Care22Answers

with MRSA activity. Cefepime and vancomycin meet these criteria (Answer D is correct). Cefepime added to genta-micin would still have no activity against MRSA (Answer B is incorrect). Answer C is incorrect because ceftriaxone is not an antipseudomonal β-lactam, and the empiric reg-imen should include two agents with activity against P. aeruginosa.1. Mandell LA, Wunderink RG, Anzueto A, Bartlett JG, Campbell

GD, Dean NC, et al. Infectious Diseases Society of America/American Thoracic Society consensus guidelines on the man-agement of community-acquired pneumonia in adults. Clin Infect Dis 2007;44(suppl 2):S27–S72.

[PubMed Link]2. Shorr AF, Owens RC Jr. Quality pneumonia care: distinguishing

community-acquired from health care-associated pneumonia. Am J Health Syst Pharm 2009;66(suppl 4):S8–S14.

[PubMed Link]

Questions 48–50 pertain to the following case.C.B. is a 71-year-old man who presents to the ED with complaints of fatigue, unsteady gait, and new abdominal pain. For the past 24 hours, C.B. found it increasingly difficult to stand because of diffuse arthralgias and myalgias. He was oriented and had five episodes of nonbloody, watery diarrhea and minimal urine output during the same period. His medical history is significant for hypertension, diabetes mellitus, obstructive sleep apnea, gout (recent exacerbation managed with systemic steroids), ventral hernia repair 1 month ago, and a recent upper respiratory infection for which he took a 7-day course of levofloxacin. Vital signs on presentation include temperature 97.5°F (36.4°C), pulse rate 96 beats/minute, respiratory rate 24 breaths/ minute, blood pressure 70/40 mm Hg, and weight 125 kg. Laboratory values include WBC count 4.9 × 103/ mm3, hematocrit 40.4%, platelet count 69,000/mm3, blood urea nitrogen 76 mg/dL, serum creatinine 7.22 mg/dL, glucose 373 mg/dL, troponin I 0.10 ng/mL, lactate 7.11 mmol/L, aspartate aminotransferase 199 units/L, alanine transaminase 203 units/L, alkaline phosphatase 433 units/L, and total bilirubin 15.4 mg/dL. Initial chest radiography is significant for mild pulmonary edema and likely small bilateral pleural effusions. Abdominal/pelvic computed tomography (CT) is ordered to evaluate for intra-abdominal inflammation and/or infection. Three liters of normal saline boluses are initiated, central and arterial catheter placement is performed, and blood cultures are obtained.

48. Which one of the following intravenous therapies is the best empiric antimicrobial management strat-egy for C.B.?A. Ertapenem, vancomycin, and metronidazole.B. Imipenem/cilastatin and fluconazole.C. Piperacillin/tazobactam and vancomycin.D. Cefepime, metronidazole, and fluconazole.

48. Answer: DTimely antimicrobial therapy is an important predictor of patient outcome and should not be withheld until diagnos-tic data are reported. The likely source of infection in this patient is the abdomen (given his clinical presentation and laboratory values), and the choice of antimicrobial therapy should have activity toward potential enteric pathogens. The patient was recently on a course of antibiotics (levo-floxacin) for presumed bronchitis, which puts him at risk of antibiotic-resistant pathogens and Clostridium difficile–associated diarrhea (the patient reports five episodes of nonbloody, watery diarrhea). The most appropriate regi-men is cefepime, metronidazole, and fluconazole because this covers for enteric gram-negative, anaerobic and fungal pathogens in addition to C. difficile (Answer D is correct). Ertapenem does not have antipseudomonal activity (Answer A is incorrect). Imipenem/cilastatin and piper-acillin/tazobactam and vancomycin do not cover for C. difficile (Answer B and Answer C are incorrect).1. Solomkin JS, Mazuski JE, Baron EJ, Sawyer RG, Nathens AB,

DiPiro JT, et al. Guidelines for the selection of anti-infective agents for complicated intra-abdominal infections. Clin Infect Dis 2003;37:997–1005.

[PubMed Link]2. Solomkin JS, Mazuki JE, Bradley JS, Rodvold KA, Goldstein

EJ, Baron EJ, et al. Diagnosis and management of complicated intra-abdominal infection in adults and children: guidelines by the Surgical Infection Society and the Infectious Diseases Society of America. Clin Infect Dis 2010;50:133–64.

[PubMed Link]3. Kumar A, Roberts D, Wood KE, Light B, Parillo JE, Sharma

S, et al. Duration of hypotension before initiation of effective antimicrobial therapy is the critical determinant of survival in human septic shock. Crit Care Med 2006;34:1589–96.

[PubMed Link]

49. C.B. remains hypotensive with a MAP of 55 mm Hg after initial fluid resuscitation (CVP 7 mm Hg). Norepinephrine is initiated, and an ScvO2 of 58% is obtained. Which of the following interventions is best to restore tissue perfusion in C.B.?A. Add dobutamine.B. Administer a crystalloid bolus.C. Transfuse 2 units of packed red blood cells.D. Add vasopressin.

49. Answer: BThe first maneuver after obtaining an ScvO2 below the target of 70% should be to assess the patient’s intravascular vol-ume status. If the CVP is abnormally low at the same time as a low ScvO2, further fluid administration with crystalloids or colloids should ensue, so 1000 mL of lactated Ringer’s is the best intervention (Answer B is correct). Administration of an inotrope such as dobutamine before adequate fluid

PSAP-VII • Critical and Urgent Care 23 Answers

resuscitation could pose a safety risk by inducing arrhyth-mia or possibly exacerbating hypotension (Answer A is incorrect). Packed red blood cells would be administered after fluid resuscitation is optimized (CVP above 8 mm Hg), ScvO2 is less than 70%, and hematocrit is less than 30%. Because the patient’s baseline hematocrit was 40.4%, he is not a candidate for red blood cell transfusion(s) at this time (Answer C is incorrect). The addition of vasopressin to norepinephrine is not indicated at this time without first failing to meet goals associated with volume resuscitation (Answer D is incorrect).1. Rivers E, Nguyen B, Havstad S, Ressler J, Muzzin A, Knoblich

B, et al. Early goal-directed therapy in the treatment of severe sepsis and septic shock. N Engl J Med 2001;345:1368–77.

[PubMed Link]2. Hollenberg SM, Ahrens TS, Annane D, Astiz ME, Chalfin DB,

Dasta JF, et al. Practice parameters for hemodynamic sup-port of sepsis in adult patients: 2004 update. Crit Care Med 2004;32:1928–48.

[PubMed Link]

50. After initiation of antimicrobial and early goal-directed therapy in the ED, C.B. is transferred to the medi-cal ICU. During the next 4 hours, his norepinephrine requirement escalates to 85 mcg/minute, and dobuta-mine (2.5 mcg/kg/minute) is initiated. Hemodynamic monitoring reveals a CVP of 15 mm Hg and ScvO2of 72%. C.B. is intubated and placed on mechanical ven-tilation with a resulting partial pressure of arterial oxygen of 98 mm Hg on 100% oxygen. New labora-tory data reveal a WBC count of 18.3 × 103 cells/mm3, hematocrit 31.4%, platelet count 15,000/mm3, and international normalized ratio 1.85. C.B. now has a temperature of 101.1°F (38.4°C). The abdominal CT is negative for ischemic colitis and abscess formation. Given his history and current clinical status, which of the following interventions is best for C.B.?A. Vasopressin 0.01 unit/minute.B. DAA 24 mcg/kg/hour.C. Epinephrine 0.05 mcg/kg/minute.D. Hydrocortisone 50 mg intravenously every 6

hours.

50. Answer: DDrotrecogin alfa (activated) is suggested therapy for patients with multisystem organ failure; however, the risk of severe bleeding in patients with a platelet count less than 30,000/mm3 may outweigh the potential benefit (Answer B is incorrect). The addition of vasopressin may be considered; however, this may offer no outcome benefit compared with norepinephrine alone (Answer A is incor-rect). Addition of epinephrine to a patient who is already taking norepinephrine and dobutamine is unnecessary and may be harmful (Answer C is incorrect). Administration of

hydrocortisone is a reasonable option in volume-refractory, vasopressor-dependent septic shock (Answer D is correct).1. Russell JA, Walley KR, Singer J, Gordon AC, Hebert PC,

Cooper DJ, et al. Vasopressin versus norepinephrine infusion in patients with septic shock. N Engl J Med 2008;358:877–87.

[PubMed Link]2. Annane D, Vignon P, Renault A, Bollaert PE, Charpentier C,

Martin C, et al. Norepinephrine plus dobutamine versus epi-nephrine alone for management of septic shock: a randomised trial. Lancet 2007;370:676–84.

[PubMed Link]

Questions 51 and 52 pertain to the following case. S.P. is a 38-year-old woman admitted to the ICU with intense abdominal pain. Her medical history is significant for end-stage liver disease secondary to alcoholic cirrhosis, grade II esophageal varices, and recurrent upper gastrointestinal bleeding events with the most recent episode 3 months ago. Her vital signs on admission to the ICU include a temperature of 99.1°F (37.3°C), respiratory rate of 20 breaths/minute, SBP of 94–98 mm Hg, and pulse rate of 92–116 beats/minute. S.P.’s initial laboratory values include WBC count 13.4 × 103 cells/mm3, lactate 7.2 mmol/L, and hematocrit 26%. Abdominal radiography is negative for bowel perforation (no free air), and the plan is to perform a paracentesis to evaluate for spontaneous bacterial peritonitis.

51. Which of the following intravenous therapies is best for S.P.?A. Dobutamine 2.5 mcg/kg/minute.B. Normal saline 20-mL/kg bolus.C. Norepinephrine 0.1 mcg/kg/minute.D. Hetastarch 500-mL bolus.

51. Answer: BAdministration of normal saline as a 20-mL/kg bolus is the best therapy because this patient may have severe sepsis. Severe sepsis is defined by two of four criteria for systemic inflammatory response syndrome (white blood cell [WBC] count of more than 12 × 103/mm3 and pulse rate greater than 90 beats/minute), a suspected infection (abdomi-nal pain and elevated WBC count), and lactate above 4 mmol/L (Answer B is correct). She may have an elevated lactate (7.2 mmol/L) secondary to her end-stage liver dis-ease; however, her elevated WBC count and low SBP are a cause for concern with sepsis. Initiation of dobutamine is not appropriate because ScvO2 is not known (Answer A is incorrect). Administration of hetastarch is controversial, especially because the patient has a baseline coagulopathy and a history of recent gastrointestinal bleeding (Answer D is incorrect). Initiation of norepinephrine is not the best choice because the patient has SBP measurements greater than 90 mm Hg and has not yet received fluid resuscitation (Answer C is incorrect).

PSAP-VII • Critical and Urgent Care24Answers

1. Rivers E, Nguyen B, Havstad S, Ressler J, Muzzin A, Knoblic B, et al. Early goal-directed therapy in the treatment of severe sep-sis and septic shock. N Engl J Med 2001;345:1368–77.

[PubMed Link]2. Finfer S, Bellomo R, Boyce N, French J, Myburgh J, Norton R.

SAFE Study Investigators. A comparison of albumin and saline for fluid resuscitation in the intensive care unit. N Engl J Med 2004;350;2247–56.

[PubMed Link]

52. S.P. has hematemesis; her repeat laboratory values are hemoglobin 7 g/dL, hematocrit 21%, and lactate 11.1 mmol/L. Her vital signs include pulse rate 127 beats/ minute, respiratory rate 28 breaths/minute, and blood pressure 90/52 mm Hg. In addition, she has become increasingly confused and has been anuric since arriv-ing at the ICU 5 hours ago. In addition to the large-bore peripheral intravenous catheter already placed, a cen-tral intravenous catheter is inserted in the internal jugular vein. Given the acute decline in hematocrit, the physician orders packed red blood cells for a pre-sumed upper gastrointestinal bleed. The microbiology laboratory reports that initial blood cultures show gram-negative rods, and broad-spectrum antibiotics are ordered. Which of the following is the best step to take in a severe sepsis protocol for S.P.?A. Repeat lactate measurement.B. Initiate normal saline 125 mL/hour.C. Determine CVP and ScvO2.D. Initiate norepinephrine 0.05 mcg/kg/minute.

52. Answer: CDetermination of the CVP and Scvo2 is the best step to take to determine whether this patient needs crystalloid or col-loid administration (Answer C is correct). The patient has not had any change in vital signs necessitating the need for norepinephrine (Answer D is incorrect). Normal saline 125 mL/hour is incorrect because crystalloids for fluid resuscitation in severe sepsis and septic shock should be administered as a bolus over 15–30 minutes (Answer B is incorrect). The patient’s serum lactate concentration is already elevated, and repeat measurements would only be indicated if an intervention aimed at changing that were implemented (Answer A is incorrect). In addition, this patient’s poor liver function may impair her ability to clear lactate, thus making it a less useful end point to monitor.1. Dellinger RP, Levy MM, Carlet JM, Bion J, Parker MM,

Jaeschke R, et al. Surviving Sepsis Campaign: international guidelines for management of severe sepsis and septic shock. Crit Care Med 2008;36:296–327.

[PubMed Link]2. SurvivingSepsis.org [homepage on the Internet]. Mount

Prospect, IL: Society of Critical Care Medicine. Available at www.survivingsepsis.org/Bundles/Pages/SepsisResusciation Bunle.aspx. Accessed October 1, 2009.

53. N.O. is a 63-year-old man with a history of multiple myeloma, atrial fibrillation, coronary artery disease, hypertension, and type 2 diabetes mellitus. He is in the ICU and has been given a diagnosis of intra-abdominal sepsis. He received all the standard therapies for sep-sis, including of all early goal-directed therapy goals and appropriate and timely antibiotics. However, he continues to deteriorate clinically. The attending phy-sician would like your opinion on any experimental therapies that might be given to N.O., preferably evi-dence-based therapies. Which of the following is the best suggestion?A. Lovastatin.B. Eritoran.C. Polymyxin B hemoperfusion.D. Recombinant bactericidal permeability protein

inhibitor.

53. Answer: CPolymyxin B hemoperfusion has been evaluated in a ran-domized, controlled trial in intraabdominal infections (Answer C is correct). Statins have only been studied in ret-rospective analyses (Answer A is incorrect). Eritoran may be an option; however, the agent is associated with atrial fibrillation during clinical trials and is thus not appropri-ate for this patient (Answer B is incorrect). Recombinant bactericidal permeability protein inhibitor has only been evaluated in pediatric meningococcal sepsis and therefore is not evidence based for this patient (Answer D is incorrect).1. LaRosa SP, Opal SM. Sepsis strategies in development. Clin

Chest Med 2008;29:735–47, x-xi. [PubMed Link]2. Cruz DN, Antonelli M, Fumagalli R, Foltran F, Brienza N,

Donati A, et al. Early use of polymyxin B hemoperfusion in abdominal septic shock: the EUPHAS randomized controlled trial. JAMA 2009;301:2445–52.

[PubMed Link]

Questions 54 and 55 pertain to the following case. H.P. is a 27-year-old woman with a history of intravenous drug use and recurrent urinary tract infections. She presented to the ED 72 hours ago with a chief complaint of dysuria, was given a dose of ciprofloxacin 400 mg intravenously, and was sent home with a prescription for ciprofloxacin 500 mg orally two times/day for 14 days. H.P. returns to the ED today with continued dysuria and new-onset light-headedness. She was unable to fill her recent antibiotic prescription because of a lack of funds and admits using an unknown oral antibiotic from her medicine cabinet for the past 2 days. Vital signs include blood pressure 92/68 mm Hg, pulse rate 128 beats/minute, respiratory rate 28 breaths/minute, and temperature 103.6°F (39.8°C). Urine microscopy is significant for pyuria (WBC more than 50/high-powered field) and 4+ bacteria. A point-of-care

PSAP-VII • Critical and Urgent Care 25 Answers

lactate is 4.6 mmol/L. Additional laboratory values and microbiologic cultures are pending.

54. Which of the following is the best empiric intra-venous anti-infective regimen for H.P.’s presumed urinary tract infection?A. Ciprofloxacin and vancomycin.B. Ceftriaxone and ciprofloxacin.C. Vancomycin and ceftriaxone.D. Meropenem, gentamicin, and vancomycin.

54. Answer: DThis patient has a history of recurrent urinary tract infec-tions and received ciprofloxacin as recently as 72 hours before admission in addition to an unknown oral antibiotic at home. Initial blood cultures have three of three bottles positive for gram-negative bacilli, and urinalysis shows gram-negative bacilli. Recent antibiotic administration and the patient’s history of intravenous drug use put her at risk of harboring resistant bacteria, and empiric therapy should be directed to target both resistant gram-positive and gram-negative pathogens. The initiation of an appro-priate antimicrobial regimen in patients with severe sepsis and septic shock decreases mortality up to 5-fold com-pared with initiation of inappropriate therapy. An empiric regimen consisting of a broad-spectrum β-lactam in com-bination with an aminoglycoside or fluoroquinolone for a gram-negative bacteremia is appropriate for this patient. Given recent fluoroquinolone therapy, concern for resis-tance to ciprofloxacin should be considered, and selection of gentamicin is prudent. Given her intravenous drug use, an agent to cover resistant gram-positive pathogens such as MRSA should be considered empirically as well. Answer D is the only regimen that fulfills these criteria and is the correct option. Ciprofloxacin plus vancomycin is incor-rect because it does not contain a second agent for resistant gram-negative bacteria and does not escalate the ciproflox-acin administered 72 hours earlier (Answer A is incorrect). Answer B is incorrect because they do not contain an anti-microbial drug to target resistant gram-positive pathogens such as MRSA. Answer C is incorrect because ceftriax-one does not provide coverage of Pseudomonas aeruginosa, a likely pathogen in this patient.1. Kumar A, Ellis P, Arabi Y, Roberts D, Light B, Parillo JE, et

al. Initiation of inappropriate antimicrobial therapy results in a fivefold reduction of survival in human septic shock. Chest 2009;136:1237–48.

[PubMed Link]2. Al-Hasan MN, Wilson JW, Lahr BD, Thomsen KM, Eckel-

Passow JE, Vetter EA, et al. Beta-lactam and fluoroquinolone combination antibiotic therapy for bacteremia caused by gram-negative bacilli. Antimicrob Agents Chemother 2009;53:1386–94.

[PubMed Link]

55. As part of H.P.’s initial laboratory work-up in the ED, a serum procalcitonin concentration is less than 0.1 mcg/L. Which of the following courses of action and justifications is best, given H.P.’s serum procal-citonin measurement?A. Discontinue antibiotics because a procalcitonin

concentration of 0.1 mcg/L or less indicates a viral infection or inflammatory response.

B. Discontinue antibiotics and reinitiate if the procalcitonin concentration rises to 0.5 mcg/L or greater.

C. Continue antibiotics because the procalcitonin concentration value represents a false negative secondary to prior antimicrobial therapy.

D. Continue antibiotics because the elevated lactate, in conjunction with procalcitonin, is diagnostic of severe sepsis.

55. Answer: CThe most correct statement is that the procalcitonin value is likely low secondary to prior antimicrobial therapy (Answer C is correct). Following serial procalcitonin concentrations may help guide antimicrobial duration in septic patients, but additional research is needed to determine more on this role. Procalcitonin, a biomarker of inflammation in bacterial infections, has varying degrees of sensitivity and specific-ity for sepsis. The patient’s procalcitonin concentration was less than 0.1 mcg/L; however, the value diagnostic of sep-sis varies between studies. Most experts agree that a value below 0.25 mcg/L is less indicative of a bacterial infection and possibly represents a viral infection or inflammatory response (Answer A is incorrect). Procalcitonin should not be used alone as a marker for sepsis, and the clinical cir-cumstances need to be considered with the interpretation of this value (Answer B is incorrect). Lactate and procalci-tonin, either alone or in combination, are not diagnostic of sepsis, and each may be elevated for noninfectious etiolo-gies (Answer D is incorrect).1. Tang BM, Eslick GD, Craig JC, McLean AS. Accuracy of procal-

citonin for sepsis diagnosis in critically ill patients: a systematic review and meta-analysis. Lancet Infect Dis 2007;7:210–7.

[PubMed Link]2. Becker KL, Snider R, Nylen ES. Procalcitonin assay in systemic

inflammation, infection, and sepsis: clinical utility and limita-tions. Crit Care Med 2008;36:941–52.

[PubMed Link]

56. N.M. is a 34-year-old, well-nourished woman (weight 87 kg) with no significant medical history except a recent dental abscess, for which she received a course of amoxicillin/clavulanic acid and no outpatient drugs. She comes to the ED with a 2-day history of fever (temperatures greater than 102.2°F [39°C]), leth-argy, abdominal pain, and persistent diarrhea. Her

PSAP-VII • Critical and Urgent Care26Answers

respiratory rate is 26 breaths/minute, and her pulse rate is above 100 beats/minute. Initial MAP is 60 mm Hg, and laboratory values are WBC count 13.2 × 103 cells/ mm3, lactate 2.8 mmol/L, blood urea nitrogen 86 mg/ dL, and serum creatinine 2.4 mg/dL. She is given 2 L of lactated Ringer’s solution over 30 minutes and maintains a MAP of 68 mm Hg for 4 hours. Which of the following best represents N.M.’s stage of sepsis?A. Severe sepsis.B. Septic shock.C. Systemic inflammatory response syndrome

(SIRS).D. Sepsis.

56. Answer: AThis patient meets criteria for systemic inflammatory response syndrome (elevated temperature and WBC count) and sepsis (infection); however, she also has evi-dence of hypoperfusion (serum lactate 2.8 mmol/L) and organ dysfunction (elevated serum creatinine) (Answer C and Answer D are incorrect). These factors make severe sep-sis the best representation of N.M’s stage of sepsis (Answer A is correct). Septic shock is incorrect because N.M. has not responded to fluid resuscitation and is not hypotensive (Answer B is incorrect).1. Marik PE, Lipman J. The definition of septic shock: implica-

tions for treatment. Crit Care Resusc 2007;9:101–3. [PubMed Link]2. Bone RC, Balk RA, Cerra FB, Dellinger RP, Fein AM, Knaus

WA, et al. Definitions for sepsis and organ failure and guide-lines for the use of innovative therapies in sepsis. The ACCP/SCCM Consensus Conference Committee. American College of Chest Physicians/Society of Critical Care Medicine. Chest 1992;101:1644–55.

[PubMed Link]

Questions 57 and 58 pertain to the following case. R.C. is a 42-year-old man who traveled to Mexico during the H1N1 influenza outbreak. He returned to your community from the trip 2 days ago and began feeling progressively ill. Worried that he may have contracted H1N1 influenza, he arrives at the ED and is placed in isolation. Once there, he becomes short of breath, his blood pressure declines, and he begins to feel light-headed. The resident in the ED activates the severe sepsis bundle; however, given the high probability of H1N1 influenza, the resident forgoes all cultures and empiric antimicrobial drugs and orders supportive care with adequate fluid resuscitation alone. 57. Which of the following is the best reason to follow

the severe sepsis protocol in R.C.?A. Protocols are meant for use in all patients; once

implemented, the bundle ensures consistently delivered processes of care that can be sustained over time.

B. Compliance with an all-or-none approach is recommended by the Surviving Sepsis Campaign and the Institute for Healthcare Improvement.

C. Bundle use with all-or-none compliance has been associated with decreases in ICU length of stay and hospital mortality.

D. All-or-none compliance is essential to decrease mortality and costs.

57. Answer: DAll-or-none compliance has been shown to decrease mortality and costs for patients in septic shock, as evidenced by numerous publications (Answer D is correct). This was most apparent in the Surviving Sepsis Campaign article published in 2010 that showed length of stay, mortality, and cost reductions, although this is generally in the setting of low compliance with the bundles. Compliance with the all-or-none approach to sepsis bundles recommended by the Surviving Sepsis Campaign and Institute for Healthcare Improvement is strongly encouraged but not required for these positive outcomes (Answer B is incorrect). Individual institutions may adopt a modified bundle without all the elements that suits local needs better and can still improve outcomes. Protocols are ideally used in most patients; however, a large quality improvement program in Spain showed that once education interventions are ceased, compliance often is not sustainable without reeducation on a regular basis as shown by the Edusepsis report (Answer A is incorrect). All-or-none compliance has been shown to decrease intensive care unit (ICU) length of stay; however, protocol implementation is not necessarily the only means to decrease ICU length of stay for all patients (Answer C is incorrect).1. Gao F, Melody T, Daniels DF, Giles S, Fox S. The impact of

compliance with 6-hour and 24-hour sepsis bundles on hos-pital mortality in patients with severe sepsis: a prospective observational study. Crit Care 2005;9:R764–70.

[PubMed Link]2. Ferrer R, Artigas A, Levy MM, Blanco J, Gonzales-Diaz G,

Garnacho-Montero J, et al. Improvement in the process of care and outcome after a multicenter severe sepsis educational pro-gram in Spain. JAMA 2008;299:2294–303.

[PubMed Link]3. Shorr AF, Micek ST, Jackson WL Jr, Kollef MH. Economic impli-

cations of an evidence-based sepsis protocol: can we improve outcomes and lower costs? Crit Care Med 2007;35:1416–7.

[PubMed Link]4. Levy MM, Dellinger RP, Townsend SR, Linde-Zwirble WT,

Marshall JC, Bion J, et al. The Surviving Sepsis Campaign: results of an international based performance improvement pro-gram targeting severe sepsis. Crit Care Med 2010;38:367–74.

[PubMed Link]

58. On questioning, R.C.’s wife notes that he began having diarrhea in Mexico and that, since returning home, he

PSAP-VII • Critical and Urgent Care 27 Answers

has had one or two episodes with blood present; oth-erwise, he is hemodynamically stable. She states that R.C. bought some antibiotics in Mexico to “pretreat for bugs in the water.” Which of the following is the most important next step for R.C.?A. Initiate fluid resuscitation.B. Initiate loperamide 2 mg orally every 8 hours.C. Obtain stool cultures.D. Initiate metronidazole 500 mg intravenously

every 8 hours.

58. Answer: CThis patient may have several etiologies responsible for his presenting symptoms. Because he recently bought unknown antibiotics for pretreatment while visiting Mexico, and may be at risk of C. difficile, the first thing that should be done is to obtain stool cultures for evaluation (Answer C is correct). However, obtaining cultures should not delay the administration of antibiotics. Metronidazole 500 mg intravenously every 8 hours is incorrect because it would be ideal to isolate a pathogen with culture before antimi-crobial therapy to help guide therapy in the future (Answer D is incorrect). Supportive care with adequate fluid resus-citation should continue; however, this answer is incorrect because fluid resuscitation alone is not sufficient (Answer A is incorrect). Initiation of loperamide in a patient at risk of C. difficile may worsen C. difficile–associated diarrhea and should not be considered (Answer B is incorrect).1. Bignardi GE. Risk factors for Clostridium difficile infection. J

Hosp Infect 1998;40:1–15. [PubMed Link]2. Kumar A, Roberts D, Wood KE, Light B, Parrillo JE, Sharma

S, et al. Duration of hypotension before initiation of effective antimicrobial therapy is a critical determinant of survival in human septic shock. Crit Care Med 2006;34:1589–96.

[PubMed Link]

Questions 59 and 60 pertain to the following case. D.K. is a 39-year-old woman who was admitted to the ICU directly from an outside hospital. She presented to the first hospital’s ED with complaints of painful burning on urination, fever for 2–3 days, and fatigue. She has a medical history of diabetes mellitus, congestive heart failure (ejection fraction of 30%), COPD, and morbid obesity (weight 220 kg). A peripheral intravenous line was inserted in the ED with normal saline initiated at 100 mL/hour. On admission to the ICU, she had a temperature of 101.7°F (38.7°C) and was intubated for hypercapnic respiratory failure. Initial laboratory values included WBC count 13.2 × 103 cells/mm3, lactate 3.1 mmol/L, INR 3.2, platelets 15,000/ mm3 and hematocrit 32%. Broad-spectrum antibiotics were initiated while central line access was obtained. By the completion of central line placement, 6 L of normal saline bolus had been administered, and norepinephrine (16 mcg/minute) was

initiated to maintain MAP of 55–65 mm Hg. D.K.’s current CVP is 14 mm Hg and ScvO2 is 68%.

59. Which of the following is the best therapy for man-aging D.K.’s septic shock?A. DAA 24 mg/kg/hour.B. Vasopressin infusion 0.01 unit/minute.C. 500 mL of normal saline every 30 minutes until

CVP reaches 15 mm Hg.D. Dobutamine infusion 2.5 mcg/kg/minute.

59. Answer: DDobutamine is the best course of action given that the patient’s ScvO2 remains below target and her hematocrit is 32%, thus making blood transfusion an inappropriate option to raise her ScvO2 (Answer D is correct). Drotrecogin alpha is incorrect because her elevated INR and low platelet count make bleeding a significant risk (Answer A is incor-rect). Vasopressin is incorrect because studies have shown that adding vasopressin to norepinephrine does not change outcomes compared with continuing norepinephrine alone (Answer B is incorrect). Because the patient’s current CVP is 14 with the target of 12–15 mm Hg for ventilated patients, further elevation by fluid bolus in a patient with an ejection fraction of 30% is unlikely to offer additional ben-efits (Answer C is incorrect).1. Dellinger RP, Levy MM, Carlet JM, Bion J, Parker MM,

Jaeschke R, et al. Surviving Sepsis Campaign: international guidelines for management of severe sepsis and septic shock. Crit Care Med 2008;36:296–327.

[PubMed Link]2. Bendjelid K, Romand JA. Fluid responsiveness in mechani-

cally ventilated patients: a review of indices used in intensive care. Intensive Care Med 2003;29:352–60.

[PubMed Link]

60. D.K. is now hemodynamically stable and has been initiated on broad-spectrum antibiotics and a ventila-tor strategy to maintain goal plateau pressures, as well as sliding-scale insulin for glucose control, famoti-dine for stress ulcer prophylaxis, and unfractionated heparin three times/day for deep venous thrombo-sis prophylaxis. The medical team also plans to begin DAA. Which of the following is the best recom-mendation regarding D.K.’s current regimen for deep venous thrombosis prophylaxis during DAA therapy?A. Change to enoxaparin.B. Change to intermittent pneumatic compression

device.C. Continue subcutaneous heparin.D. Reduce subcutaneous heparin to two times/day.

60. Answer: C

PSAP-VII • Critical and Urgent Care28Answers

The role of initiation or continuation of prophylactic hep-arin was studied by the Xigris and Prophylactic HepaRin Evaluation in Severe Sepsis (XPRESS) study group. In a subgroup analysis, patients exposed to heparin at baseline and then randomized to placebo had a higher mortal-ity than exposed patients subsequently randomized to the heparin group, so continuation of prophylactic heparin is recommended. Because this patient was initiated on subcu-taneous heparin before DAA was started, the best answer is to continue heparin (Answer C is correct). It is not rec-ommended to stop subcutaneous heparin because this may put the patient at a higher risk of developing deep venous thrombosis despite DAA therapy (Answer B is incorrect). Changing to a low-molecular-weight heparin is unnec-essary and problematic, as shown in the XPRESS study (Answer A is incorrect). The dose of heparin is not likely an issue because heparin 15,000 units/day was allowed in the PROWESS trial (Answer D is incorrect).1. Levi M, Levy M, Williams MD, Douglas I, Artigas A, Antonelli

M, et al. Xigris and Prophylactic HepaRin Evaluation in Severe Sepsis (XPRESS) Study Group. Prophylactic heparin in patients with severe sepsis treated with drotrecogin alfa (acti-vated). Am J Respir Crit Care Med 2007;176:483–90.

[PubMed Link]2. Shorr AF, Williams MD. Venous thromboembolism in crit-

ically ill patients. Observations from a randomised trial in sepsis. Thromb Haemost 2009;101:139–44.

[PubMed Link]

Thrombotic and Bleeding Disorders in the Critically Ill

61. N.D. is a 75-year-old woman (weight 45 kg, height 5′1′′, serum creatinine 0.6 mg/dL) admitted to the medical-surgical intensive care unit (ICU) for an acute exacerbation of chronic obstructive pulmonary dis-ease. She has a 60 pack-year history of smoking and has hypertension and hypothyroidism. What thrombo-prophylaxis (TP) is most appropriate for N.D.?A. Enoxaparin 40 mg subcutaneously once daily.B. Unfractionated heparin (UFH) 5000 units

subcutaneously three times/day.C. Enoxaparin 30 mg subcutaneously two times/day.D. UFH 5000 units subcutaneously two times/day.

61. Answer: DEnoxaparin and other low-molecular-weight heparins (LMWHs) have not been shown superior to unfraction-ated heparin (UFH) in general surgical and medical intensive care unit (ICU) patients (Answer A and Answer C are incorrect). Unfractionated heparin three times/day is a valid option, but considering the low weight and age of this patient, the bleeding risk outweighs the potential ben-efits (Answer B is incorrect). Therefore, the same dose two times/day is the best option (Answer D is correct).1. Geerts WH, Bergqvist D, Pineo GF, Heit JA, Samama CM,

Lassen MR, et al. Prevention of venous thromboembolism: American College of Chest Physicians Evidence-Based Clinical Practice Guidelines (8th ed). Chest 2008;133:381S–453S.

[PubMed Link]2. King CS, Holley AB, Jackson JL, Shorr AF, Moores LK. Twice

vs three times daily heparin dosing for thromboembolism pro-phylaxis in the general medical population: a meta-analysis. Chest 2007;131:507–16.

[PubMed Link]

62. T.W. is a 78-year-old man (weight 68 kg, height 5′10′′) transferred to the surgical ICU from the operating room after resection of an adenocarcinoma of the colon. His medical history includes complicated dia-betes with secondary chronic kidney insufficiency (serum creatinine 1.4 mg/dL). He is initiated on sub-cutaneous enoxaparin 30 mg two times/day on the day after surgery. During his ICU stay, he underwent radioactive contrast imaging to rule out mesenteric ischemia. In the days after the examination, his serum creatinine increased considerably (now 2.4 mg/dL). Which of the following recommendations is best for T.W. regarding his enoxaparin therapy?A. Continue enoxaparin 30 mg subcutaneously

twice daily.B. Reduce enoxaparin to 40 mg subcutaneously

once daily.C. Discontinue enoxaparin and begin heparin

subcutaneously 5000 units two times/day.D. Discontinue enoxaparin and begin dalteparin

5000 units once daily.

62. Answer: DEnoxaparin accumulates in patients with severe kidney failure (creatinine clearance [CrCl] less than 30 mL/min-ute). The manufacturer suggests reducing the dose to 30 mg/day in patients with severe kidney failure. Answer A and Answer B are not consistent with these recommenda-tions and are therefore incorrect. Although heparin would be a good alternative in the general ICU population, the history of adenocarcinoma warrants the use of an LMWH (Answer C is incorrect). Dalteparin 5000 international units once daily was recently shown in a prospective study

PSAP-VII • Critical and Urgent Care 29 Answers

(DIRECT study) not to accumulate in critically ill patients with severe kidney failure. Given the patient’s history of cancer, dalteparin is the most appropriate alternative (Answer D is correct).1. Geerts WH, Bergqvist D, Pineo GF, Heit JA, Samama CM,

Lassen MR, et al. Prevention of venous thromboembolism: American College of Chest Physicians Evidence-Based Clinical Practice Guidelines (8th ed). Chest 2008;133:381S–453S.

[PubMed Link]2. Douketis J, Cook D, Meade M, Guyatt G, Geerts W, Skrobik

Y. Prophylaxis against deep vein thrombosis in critically ill patients with severe renal insufficiency with the low-molec-ular-weight heparin dalteparin: an assessment of safety and pharmacodynamics: the DIRECT study. Arch Intern Med 2008;168:1805–12.

[PubMed Link]

63. A.P. is a 23-year-old woman involved in a motor vehicle crash. She suffered several fractures to both tibias that were reduced with external fixators on day 1 of admis-sion. Free fluid in the abdominal space was found on focused abdominal sonography for trauma in the emergency department, warranting urgent surgery. A rupture in the fourth segment of the liver was found and surgically repaired. Hemostasis was successfully attained after a 3-hour surgery, and the patient is now admitted to your ICU. Which of the following is the best TP therapy for A.P. on day 1 of ICU admission?A. Enoxaparin 40 mg subcutaneously once daily

initiated immediately postoperatively.B. Enoxaparin 30 mg subcutaneously twice daily

initiated on day 2 postoperatively.C. Intermittent compression devices (ICDs) on both

legs initiated immediately postoperatively.D. Retractable inferior vena cava filter insertion

immediately postoperatively.

63. Answer: BGiven that the patient has fixators on both legs, intermit-tent compression devices (ICDs) will not be convenient (Answer C is incorrect). Enoxaparin should be initiated as soon as possible at the optimal dose. In orthopedic trauma, the evidence-based dosing regimen for enoxaparin is 30 mg two times/day (Answer A is incorrect). Because the patient is postoperative day 1, it is reasonable to post-pone thromboprophylaxis (TP) on day 2, considering her risk of bleeding (Answer B is correct). Data supporting the use of prophylactic inferior vena cava filters are limited, and these devices should be used only for exceptional situations when it is not possible to initiate pharmacologic TP safely (Answer D is incorrect).1. Geerts WH, Bergqvist D, Pineo GF, Heit JA, Samama CM,

Lassen MR, et al. Prevention of venous thromboembolism:

American College of Chest Physicians Evidence-Based Clinical Practice Guidelines (8th ed). Chest 2008;133:381S–453S.

[PubMed Link]2. Geerts WH, Jay RM, Code KI, Chen E, Szalai JP, Saibil EA, et al.

A comparison of low-dose heparin with low-molecular-weight heparin as prophylaxis against venous thromboembolism after major trauma. N Engl J Med 1996;335:701–7.

[PubMed Link]

64. C.T. is a 49-year-old man (CrCl 110 mL/minute) admitted for elective neurosurgical biopsy and removal of a glioblastoma. Because of the high risk of venous thromboembolism (VTE), TP is warranted. Before surgery, the neurosurgery fellow inquires about the best strategy and timing for initiating TP. Which of the following recommendations is best for C.T.?A. Enoxaparin 40 mg subcutaneously once daily

initiated preoperatively in combination with an ICD.

B. Heparin subcutaneously 5000 units two times/ day initiated postoperatively in combination with an ICD.

C. Enoxaparin 30 mg subcutaneously two times/day initiated postoperatively without an ICD.

D. Enoxaparin 40 mg subcutaneously once daily initiated postoperatively without an ICD.

64. Answer: BThe most recent American College of Chest Physicians (ACCP) guidelines for neurosurgical patients at high risk of venous thromboembolism (VTE) (such as this patient) recommend the combination of an ICD with either an LMWH or UFH administered postoperatively (Answer B is correct). Perioperative administration of enoxaparin has been associated with excessive postoperative bleeding and should be avoided (Answer A is incorrect). Numerous studies have shown the efficacy of ICDs in preventing VTEs in neurosurgical patients, and they should be used in all neurosurgical patients regardless of UFH or LMWH use (Answer C and Answer D are incorrect).1. Geerts WH, Bergqvist D, Pineo GF, Heit JA, Samama CM,

Lassen MR, et al. Prevention of venous thromboembolism: American College of Chest Physicians Evidence-Based Clinical Practice Guidelines (8th ed). Chest 2008;133:381S–453S.

[PubMed Link]2. Collen JF, Jackson JL, Shorr AF, Moores LK. Prevention of

venous thromboembolism in neurosurgery: a meta-analysis. Chest 2008;134:237–49.

[PubMed Link]

65. R.D. is a 73-year-old woman (56 kg) admitted for community-acquired pneumonia. Her medical history includes type 2 diabetes mellitus, chronic kidney fail-ure, and coronary heart disease. Her serum creatinine

PSAP-VII • Critical and Urgent Care30Answers

is 2.0 mg/dL, her urine output is normal, her hemo-globin is stable at 10 g/dL, and her platelet count is 90 × 109/L. She is mechanically ventilated and receiving the following drugs: aspirin 80 mg by nasogastric tube once daily, ranitidine 50 mg intravenously once daily, erythropoietin 3000 units subcutaneously three times/ week, norepinephrine infusion at 0.1 mcg/kg/min-ute, and piperacillin/tazobactam 2.25 g intravenously every 8 hours. Which of the following interventions is best to reduce R.D.’s risk of a VTE?A. Discontinue erythropoietin and initiate an ICD

until the platelet count is greater than 100 × 109/L.

B. Continue erythropoietin, initiate an ICD plus dalteparin 5000 units once daily, and monitor platelet counts.

C. Continue erythropoietin, initiate UFH 5000 units every 12 hours, and monitor platelet counts.

D. Discontinue erythropoietin, initiate enoxaparin 40 mg once daily, and monitor anti-Xa levels.

65. Answer: CErythropoietin for the treatment of ICU-associated anemia at doses of 40,000 international units once weekly has been associated with an increased risk of deep venous thrombo-sis; however, there are no reports of such an association in patients with chronic kidney failure receiving lower dosages of erythropoietin while in the ICU (Answer A and Answer D are incorrect). There are no data supporting the com-bined use of ICDs with pharmacologic measures to prevent VTE in critically ill medical patients (Answer B is incor-rect). In addition, in medical patients, LMWH has not been shown to be superior to UFH, and moderate thrombocy-topenia is not a contraindication to pharmacologic TP in medical patients (Answer C is correct).1. Geerts WH, Bergqvist D, Pineo GF, Heit JA, Samama CM,

Lassen MR, et al. Prevention of venous thromboembolism: American College of Chest Physicians Evidence-Based Clinical Practice Guidelines (8th ed). Chest 2008;133:381S–453S.

[PubMed Link]2. Corwin HL, Gettinger A, Fabian TC, May A, Pearl RG, Heard

S, et al. Efficacy and safety of epoetin alfa in critically ill patients. N Engl J Med 2007;357:965–76.

[PubMed Link]

66. M.T. is a 42-year-old woman admitted to the emer-gency department for shortness of breath and bilateral swelling of the lower extremities. Her blood pressure is 98/60 mm Hg, and her pulse rate is 120 beats/min-ute. Her laboratory tests show increased D-dimers and positive cardiac troponins, but otherwise, all values are within normal limits. A multidetector computerized tomography (CT) scan and leg Doppler ultrasonogra-phy reveal evidence of an acute pulmonary embolism

(PE) and bilateral deep venous thrombosis (DVT). Unfractionated heparin is initiated, and the patient is transferred to the ICU. Echocardiography reveals pulmonary hypertension as well as moderate right ventricular dysfunction. Which intervention is most appropriate to manage M.T.’s PE?A. Continue with UFH.B. Discontinue UFH and begin alteplase.C. Continue with UFH and add alteplase.D. Discontinue UFH and begin enoxaparin.

66. Answer: BThis patient does not have refractory hypotension and thus cannot be classified as having massive pulmonary embo-lism (PE). However, she has several high-risk features such as positive troponins, right ventricular enlargement, and pulmonary hypertension. The benefits of thrombolysis are greatest in the first 48 hours. This patient could bene-fit from systemic thrombolysis because of several high-risk features (Answer B is correct; Answer A and Answer D are incorrect). This is a grade 2B recommendation from the most recent ACCP guidelines. In addition, heparin should be stopped and thrombolysis started when the activated partial thromboplastin time is below 80 seconds, making Answer C incorrect.1. Kearon C, Kahn SR, Agnelli G, Goldhaber S, Raskob GE,

Comerota AJ. Antithrombotic therapy for venous throm-boembolic disease: American College of Chest Physicians Evidence-Based Clinical Practice Guidelines (8th ed). Chest 2008;133:454S–545S.

[PubMed Link]2. Konstantinides S, Geibel A, Heusel G, Heinrich F, Kasper

W; Management Strategies and Prognosis of Pulmonary Embolism-3 Trial Investigators. Heparin plus alteplase compared with heparin alone in patients with submassive pul-monary embolism. N Engl J Med 2002;347:1143–50.

[PubMed Link]

67. N.P. is a 68-year-old man with an estimated CrCl of 25 mL/minute admitted to the ICU in septic shock secondary to acute peritonitis. His medical history includes hypertension, chronic kidney failure, hyper-cholesterolemia, and hemorrhagic stroke suffered 4 months ago. He is intubated, fluid resuscitated, initi-ated on antibiotics, and sent to the operating room for resection of an ischemic colon. His postoperative med-ication orders include meropenem 1 g every 8 hours, subcutaneous heparin 5000 units two times/day, nor-epinephrine infusion at 0.12 mcg/kg/minute, and ranitidine. On postoperative day 3, he develops sud-den hypercapnia with associated hypoxia and right ventricular failure with pulmonary hypertension; the norepinephrine dose is increased to 1 mcg/kg/minute because of cardiovascular collapse. The medical team

PSAP-VII • Critical and Urgent Care 31 Answers

suspects massive PE. Which of the following inter-ventions is best for N.P. to treat his PE?A. Alteplase 10-mg bolus and 90 mg over 2 hours.B. Tenecteplase 50-mg intravenous bolus.C. UFH 80-unit/kg bolus, then 18 units/kg/hour.D. Alteplase 50 mg intravenous over 15 minutes.

67. Answer: AThe most studied agent for PE is alteplase. Although no differences have been shown between the two alteplase regimens, the body of evidence has used the 100 mg over 2 hours regimen (Answer A is correct; Answer D is incor-rect). Tenecteplase is not as well studied, making it a less ideal choice (Answer B is incorrect). Answer C is incorrect because thrombolysis is indicated for the patient. 1. Kearon C, Kahn SR, Agnelli G, Goldhaber S, Raskob GE,

Comerota AJ. Antithrombotic therapy for venous throm-boembolic disease: American College of Chest Physicians Evidence-Based Clinical Practice Guidelines (8th ed). Chest 2008;133:454S–545S.

[PubMed Link]2. Torbicki A, Perrier A, Konstantinides S, Giancarlo Agnelli,

Nazzareno Galiè, Piotr Pruszczyk, et al. Guidelines on the diagnosis and management of acute pulmonary embolism: the Task Force for the Diagnosis and Management of Acute Pulmonary Embolism of the European Society of Cardiology (ESC). Eur Heart J 2008;29:2276–315.

[PubMed Link]

68. A.F. is a 57-year-old woman admitted to the ICU for septic shock secondary to community-acquired pneumonia with multiple organ failure. She requires continuous renal replacement therapy (CRRT) for acute kidney failure and high norepinephrine doses to maintain her blood pressure. On day 1 of admission she is initiated on dalteparin 5000 units once daily. On day 3, her platelet count decreases from 220,000/mm3

to 90,000/mm3. Which of the following investiga-tions is best for A.F.?A. Heparin/PF4 antibody enzyme-linked

immunosorbent assay (ELISA) test.B. Serotonin-releasing assay.C. 4T score.D. Dual-leg Doppler ultrasonography.

68. Answer: CThe thrombocytopenia has developed too early to corre-spond with a heparin-induced thrombocytopenia (HIT) diagnosis, making HIT assays irrelevant (Answer A and Answer B are incorrect). The best clinical approach would be to calculate a 4T score to evaluate the probability of HIT (Answer C is correct). It is premature to ask for leg Doppler ultrasonography because there is no clinical sus-

picion of deep venous thrombosis or proven HIT (Answer D is incorrect).1. Warkentin TE, Greinacher A, Koster A, Lincoff AM. Treatment

and prevention of heparin-induced thrombocytopenia: American College of Chest Physicians Evidence-Based Clinical Practice Guidelines (8th ed). Chest 2008;133:340S–380S.

[PubMed Link]2. Lo GK, Juhl D, Warkentin TE, Sigouin CS, Eichler P,

Greinacher A. Evaluation of pretest clinical score (4T’s) for the diagnosis of heparin-induced thrombocytopenia in two clinical settings. J Thromb Haemost 2006;4:759–65.

[PubMed Link]

69. D.R. is a 52-year-old man admitted to the ICU after elective coronary artery bypass surgery. The same day, he is initiated on subcutaneous heparin 5000 units three times/day. His preoperative platelet count before surgery was 170 × 109/L. The platelet count after the surgery is 140 × 109/L, which drops to 110 × 109/L the next day and 90 × 109/L the day after. On day 3, the platelet count remains low (95 × 109/L), so the medi-cal resident asks for a heparin/PF4 antibody test and discontinues all heparins. Of note, the patient had not been exposed to heparin in the 6 months before sur-gery. The platelet count increases in the days after heparin is discontinued (110 × 109/L, 130 × 109/L, and 170 × 109/L). Six days after surgery, the HIT ELISA test comes back positive. Patient is now on the general surgery ward and ambulating regularly. Which of the following interventions is best for D.R.?A. Therapeutic dose of direct thrombin inhibitor.B. Perform an HIT functional assay.C. Continue to hold heparin doses.D. Reinitiated on prophylactic heparin.

69. Answer: BModerate thrombocytopenia after cardiac surgery with extracorporeal circulation is common and generally resolves within 5 days. Heparin-induced thrombocyto-penia in a patient not exposed to heparin often develops between 5 days and 14 days of heparin exposure. The inci-dence of asymptomatic heparin/PF4 antibodies in patients undergoing cardiac surgery ranges from 20% to 50%. Of these patients, about 1% have true HIT. In this case, the probability of HIT is low. A functional assay such as the serotonin assay would confirm an asymptomatic HIT anti-body that cannot induce the HIT syndrome (Answer B is correct). Treating for HIT based on this questionable posi-tive enzyme-linked immunosorbent assay (ELISA) test is not recommended (Answer A is incorrect). Although very unlikely, this may still be a true HIT, and diagnosis needs to be confirmed. Therefore, the ELISA test result cannot be ignored (Answer D is incorrect) or not acted on (Answer C is incorrect).

PSAP-VII • Critical and Urgent Care32Answers

1. Warkentin TE, Greinacher A, Koster A, Lincoff AM. Treatment and prevention of heparin-induced thrombocytopenia: American College of Chest Physicians Evidence-Based Clinical Practice Guidelines (8th ed). Chest 2008;133:340S–380S.

[PubMed Link]2. Pouplard C, Gueret P, Fouassier M, Ternisien C, Trossaert

M, Regina S, et al. Prospective evaluation of the 4T’s score and particle gel immunoassay specific to heparin/PF4 for the diagnosis of heparin-induced thrombocytopenia. J Thromb Haemost 2007;5:1373–9.

[PubMed Link]

70. A.B. has a diagnosis of three-vessel coronary artery dis-ease requiring elective bypass surgery that has been scheduled to occur in 4 weeks. The patient has a his-tory of HIT with associated thrombosis 3 months ago, diagnosed while admitted for unstable angina. Which of the following is the most appropriate plan for his surgery in view of his past HIT diagnosis?A. Perform no further HIT testing; use heparin

intraoperatively and danaparoid postoperativelyB. Perform no further HIT testing; use bivalirudin

intraoperatively and enoxaparin postoperatively.C. Perform ELISA HIT assay and, if positive,

postpone surgery.D. Perform ELISA HIT assay and, if positive, use

bivalirudin intraoperatively.

70. Answer: CIn cardiac surgery, heparin is the anticoagulant of choice and should be used when possible. However, this patient may still have positive HIT antibodies because his initial HIT was less than 100 days ago. Therefore, testing for anti-bodies is warranted before he can be reexposed to heparin (Answer A and Answer B are incorrect). When possible, postponing the surgery until ELISA is negative is recom-mended by the ACCP guidelines (Answer C is correct; Answer D is incorrect). However, if for any reason surgery is warranted earlier, use of bivalirudin intraoperatively is an acceptable alternative.1. Warkentin TE, Greinacher A, Koster A, Lincoff AM. Treatment

and prevention of heparin-induced thrombocytopenia: American College of Chest Physicians Evidence-Based Clinical Practice Guidelines (8th ed). Chest 2008;133:340S–380S.

[PubMed Link]2. Dager WE, Dougherty JA, Nguyen PH, Militello MA,

Smythe MA. Heparin-induced thrombocytopenia: treat-ment options and special considerations. Pharmacotherapy 2007;27:564–87.

[PubMed Link]

71. W.T., a 68-year-old man, was admitted to the ICU after an emergency abdominal aortic aneurysm resection.

Subcutaneous heparin was begun on postoperative day 1. On postoperative day 5, his course was compli-cated by nosocomial pneumonia with secondary septic shock and associated acute liver (peak aspartate ami-notransferase 1200 international units/mL and peak alanine aminotransferase 900 international units/mL) and kidney failure. On day 6, he became anuric, and CRRT requiring systemic anticoagulation with UFH was initiated. His platelet counts dropped from 200 × 109/L to 85 × 109/L over 48 hours. The 4T score reveals an intermediate risk of HIT, and the attend-ing physician orders a heparin/PF4 antibody test. In addition to discontinuing heparin, which of the fol-lowing strategies is best for W.T. while awaiting the result of the heparin/PF4 antibody test?A. Initiate argatroban 0.25 mcg/kg/minute.B. Initiate lepirudin 25 mg subcutaneously twice

daily.C. Initiate bivalirudin 0.14 mg/kg/hour.D. Initiate citrate anticoagulation for the CRRT.

71. Answer: ABecause this patient has an intermediate risk of HIT, a sys-temic alternative anticoagulation is warranted. Answer D is incorrect because sodium citrate will only anticoagulate the circuit. Bivalirudin would be an appropriate treatment; however, the dose proposed is too high and should be adjusted for multiple-organ failure (Answer C is incorrect). Lepirudin is not the best option because it is primarily elim-inated by the kidneys and will likely require dose reduction or complete avoidance in this patient (Answer B is incor-rect; Answer A is correct).1. Warkentin TE, Greinacher A, Koster A, Lincoff AM. Treatment

and prevention of heparin-induced thrombocytopenia: American College of Chest Physicians Evidence-Based Clinical Practice Guidelines (8th ed). Chest 2008;133:340S–380S.

[PubMed Link]2. Selleng K, Warkentin TE, Greinacher A. Heparin-induced

thrombocytopenia in intensive care patients. Crit Care Med 2007;35:1165–76.

[PubMed Link]

72. A 72-year-old man (weight 70 kg) was discharged 4 days ago after a 2-vessel coronary artery bypass. He spent 4 days in the hospital before discharge. During his stay, his platelet counts were 90 × 109/L, 160 × 109/L, 210 × 109/L, and 175 × 109/L on postoperative days 1–4, respectively. He now presents to emergency department with shortness of breath, cardiac troponin- I of 0.1 ng/mL, positive D-dimers, and platelet count of 100 × 109/L. His blood pressure is 140/80 mm Hg, and his pulse rate is 120 beats/minute. With CT angi-ography, a PE is diagnosed. Which of the following strategies is best for this patient?

PSAP-VII • Critical and Urgent Care 33 Answers

A. Initiate alteplase 10-mg bolus and 90 mg over 2 hours immediately, followed by dalteparin 100 international units/kg once daily.

B. Initiate heparin 80-international unit/kg bolus, followed by an 18-international unit/kg/hour infusion immediately.

C. Initiate fondaparinux 7.5 mg once daily immediately.

D. Initiate alteplase 10-mg bolus and 90 mg over 2 hours, immediately followed by lepirudin 0.1 mg/ kg/hour.

72. Answer: CThrombolysis is not indicated for this patient because he does not fulfill the criteria for massive or submassive PE (Answer A and Answer D are incorrect). Although cardiac troponins seem positive, this patient recently underwent cardiac bypass. Increased cardiac troponin is expected up to 10 days after surgery. His platelet count on arrival to the emergency department is 50% lower than his platelet count was during his stay. A HIT with thrombosis is there-fore likely, and heparin should be avoided (Answer B is incorrect). Fondaparinux is an appropriate treatment of HIT with or without thrombosis and of PE (Answer C is correct).1. Warkentin TE, Greinacher A, Koster A, Lincoff AM. Treatment

and prevention of heparin-induced thrombocytopenia: American College of Chest Physicians Evidence-Based Clinical Practice Guidelines (8th ed). Chest 2008;133:340S–380S.

[PubMed Link]2. Torbicki A, Perrier A, Konstantinides S, Giancarlo Agnelli,

Nazzareno Galiè, Piotr Pruszczyk, et al. Guidelines on the diag-nosis and management of acute pulmonary embolism: the Task Force for the Diagnosis and Management of Acute Pulmonary Embolism of the European Society of Cardiology (ESC). Eur Heart J 2008;29:2276–315.

[PubMed Link]

73. An 82-year-old patient is admitted on the pulmonary ward for an acute exacerbation of chronic obstructive pulmonary disease. The patient receives TP with hep-arin 5000 international units two times/day. During his stay, he develops a DVT in his right lower limb. Heparin is discontinued, and tinzaparin 175 interna-tional units/kg once daily and warfarin are initiated. Three days later, the ICU team is consulted because the patient has progressive skin necrosis of his right toes with associated severe pain. His INR is 1.9, and platelet count is 20 × 109/L. Which of the following is best to manage the patient’s condition?A. Discontinue warfarin and tinzaparin and

administer vitamin K.B. Continue warfarin, discontinue tinzaparin, and

initiate lepirudin 0.1 mg/kg/hour.

C. Continue warfarin, discontinue tinzaparin, and initiate argatroban 2 mcg/kg/minute.

D. Discontinue warfarin, administer vitamin K, and initiate argatroban 2 mcg/kg/minute.

73. Answer: DThis patient likely has warfarin-induced skin necrosis associated with HIT. Therefore, warfarin should be discon-tinued and reversed with vitamin K, as the ACCP consensus guidelines suggest (Answer B and Answer C are incorrect). Because this patient has a HIT with thrombosis, treatment is warranted (Answer A is incorrect; Answer D is correct). 1. Warkentin TE, Greinacher A, Koster A, Lincoff AM. Treatment

and prevention of heparin-induced thrombocytopenia: American College of Chest Physicians Evidence-Based Clinical Practice Guidelines (8th ed). Chest 2008;133:340S–380S.

[PubMed Link]2. Selleng K, Warkentin TE, Greinacher A. Heparin-induced

thrombocytopenia in intensive care patients. Crit Care Med 2007;35:1165–76.

[PubMed Link]

74. M.R. is a 22-year-old man involved in a motor vehicle crash. The first Glasgow Coma Scale (GCS) score at the trauma scene was 10/15. He was taken to the emer-gency department, where he was intubated before his head CT. During the intubation procedure, his blood pressure decreased to 65/40 mm Hg and his pulse rate increased to 150 beats/minute. A focused abdominal sonography for trauma was performed showing retro-peritoneal bleeding, most likely because of a kidney rupture. M.R. has received 4 units of packed red blood cells. At this point, his hemoglobin is 4.4 g/dL, his INR is 2.63, his platelet count is 130 × 109/L, and his body core temperature is 35.5°C. Which of the following strategies is best for managing M.R.’s bleeding?A. rf VIIa 100 mcg/kg.B. 2 units of packed red blood cells and rf VIIa at 20

mcg/kg.C. Prothrombin complex concentrate (PCC).D. 2 units of packed red cells and fresh frozen

plasma.

74. Answer: DThe published trials that have studied recombinant factor VIIa (rf VIIa) in the context of abdominal trauma have all used the administration of 6 units of blood as an inclusion criterion. The activity of rf VIIa may be reduced in the pres-ence of hypothermia and acidosis (Answer A and Answer B are incorrect). No randomized, controlled studies have evaluated the safety and efficacy of prothrombin complex concentrate in hemorrhagic shock (Answer C is incorrect; Answer D is correct).

PSAP-VII • Critical and Urgent Care34Answers

1. Boffard KD, Riou B, Warren B, Choong PI, Rizoli S, Rossaint R, et al. Recombinant factor VIIa as adjunctive therapy for bleeding control in severely injured trauma patients: two paral-lel randomized, placebo-controlled, double-blind clinical trials. J Trauma 2005;59:8–18.

[PubMed Link]2. Hsia CC, Chin-Yee IH, McAlister VC. Use of recombi-

nant activated factor VII in patients without hemophilia. Ann Surg 2008;248:61–8.

[PubMed Link]

75. A.B. is a 24-year-old man involved in a motor vehicle crash while intoxicated with ethanol. He is admitted to the emergency department of a trauma center. His GCS score is 6, and he is intubated. A focused abdom-inal sonography for trauma shows fluid accumulated in the retroperitoneal space. His hemoglobin is 6.8 g/dL, and he receives 2 units of red blood cells. His blood pressure is 95/60 mm Hg, and his pulse rate is 130 beats/minute. A norepinephrine infusion is initi-ated together with aggressive volume resuscitation to maintain a mean arterial pressure of 65 mm Hg. The emergency team plans to bring the patient for a CT scan after he stabilizes. Now, 3 hours after being admit-ted, he is receiving his sixth unit of blood. Which strategy is best to manage A.B.’s bleeding?A. Administer 200 mcg of rf VIIa immediately.B. Wait until the eighth unit of red blood cells has

been given, and administer 200 mcg of rf VIIa if needed.

C. Delay the administration of rf VIIa until the head CT result is available.

D. Administer 20 mcg/kg of rf VIIa immediately.

75. Answer: CStudies evaluating the use of rf VIIa in abdominal trauma patients excluded patients presenting with concomitant traumatic brain injuries (defined as an initial GCS score less than 8). Recombinant factor VIIa has not been proved effi-cacious in traumatic brain injury (Answer A, Answer B, and Answer D are incorrect). Because the patient was intoxi-cated with ethanol, his actual GCS score might not reflect his brain condition. The results of a CT scan are neces-sary to exclude traumatic brain injury before administering rf VIIa (Answer C is correct)1. Boffard KD, Riou B, Warren B, Choong PI, Rizoli S, Rossaint

R, et al. Recombinant factor VIIa as adjunctive therapy for bleeding control in severely injured trauma patients: two paral-lel randomized, placebo-controlled, double-blind clinical trials. J Trauma 2005;59:8–18.

[PubMed Link]2. Narayan RK, Maas AIR, Marshall LF, Servadei F, Skolnick

BE, Tillinger MN; rf VIIa Traumatic ICH Study Group. Recombinant factor VIIa in traumatic intracerebral hemor-rhage: results of a dose-escalation clinical trial. Neurosurgery 2008;62:776–88.

[PubMed Link]

76. R.A. is a 55-year-old man with a medical history of diabetes, stable angina, hypertension, recent proxi-mal DVT, and hypercholesterolemia. He is admitted to the ICU for decreased level of consciousness after an 11-stair fall. His GCS score on admission is 11. The patient is not intubated. A head CT scan shows a 5-mL subdural hematoma in the frontal lobe. Which course of action regarding rf VIIa administration is best for R.A.?A. Do not administer this agent.B. Administer if GCS score falls below 6.C. Administer an 80-mcg/kg dose now.D. Administer a 20-mcg/kg dose now.

76. Answer: ABecause rf VIIa has no proven clinical benefits on mortal-ity or neurologic functional status in patients with cerebral spontaneous hemorrhage, but only on hematoma growth, the decision to give rf VIIa should be made on the basis of individual patients. Hematoma size is one of the clinical features that should be considered. In this case, the small size of the hematoma and the Glasgow Coma Scale (GCS) score preclude the administration of rf VIIa (Answer C and Answer D are incorrect). If the GCS score deteriorates, a repeat CT scan should be performed before administering rf VIIa (Answer B is incorrect; Answer A is correct). 1. Mayer SA, Brun NC, Begtrup K, Broderick J, Davis S, Diringer

MN, et al. Efficacy and safety of recombinant activated fac-tor VII for acute intracerebral hemorrhage. N Engl J Med 2008;358:2127–37.

[PubMed Link]2. Hsia CC, Chin-Yee IH, McAlister VC. Use of recombinant

activated factor VII in patients without hemophilia. Ann Surg 2008;248:61–8.

[PubMed Link]

77. A 62-year-old man arrives in the ICU after a beating heart minimally invasive cardiac bypass surgery. His medical history includes diabetes, hypercholesterol-emia, total knee replacement, and heparin-induced thrombocytopenia 2 years ago. On arrival, the patient’s blood pressure decreases to 65/45 mm Hg and his pulse rate increases to 160 beats/minute. Severe bleed-ing from the graft is suspected. The patient receives fluid and vasopressor support as well as 6 units of packed red blood cells, 10 units of fresh frozen plasma, and 5 units of platelet transfusion in addition to des-mopressin. His pH is 7.28, and core body temperature is 35°C. Emergency thoracotomy is planned. Pertinent laboratory values are calcium 4.0 mg/dL, bicarbonate 25 mEq/L, and pH 7.30. Which of the following is best to administer to this patient?A. rf VIIa.B. PCC.

PSAP-VII • Critical and Urgent Care 35 Answers

C. Sodium bicarbonate and 5 units of fresh frozen plasma.

D. Sodium bicarbonate and rf VIIa.

77. Answer: AThis patient has normal bicarbonates and does not require bicarbonates to correct his pH (Answer C and Answer D are incorrect). Because the hemorrhage is not secondary to vitamin K antagonist administration, prothrombin complex concentrates are not indicated (Answer B is incorrect). The severity and refractoriness of the bleeding requires prompt treatment (Answer A is correct).1. Diprose P, Hebertson MJ, O’Shaughnessy D, Gill RS. Activated

recombinant factor VII after cardiopulmonary bypass reduces allogeneic transfusion in complex non-coronary cardiac sur-gery: randomized double-blind placebo-controlled pilot study. Br J Anaesth 2005;95:596–602.

[PubMed Link]2. Hsia CC, Chin-Yee IH, McAlister VC. Use of recombinant

activated factor VII in patients without hemophilia. Ann Surg 2008;248:61–8.

[PubMed Link]

78. A patient is admitted to a level 1 trauma center for a severe hemorrhage secondary to a splenic artery rup-ture and receives three doses of rf VIIa. The patient also has an open pelvic fracture. On day 5 of admission to the ICU, the patient develops acute shortness of breath with oxygen desaturation consistent with a diagno-sis of PE. The patient is on compressive stockings but has received no other TP since admission. Which of the following sets of risk factors best describes the potential causes of PE?A. Splenic artery rupture, pelvic fracture, no TP.B. Pelvic fracture, no TP, rf VIIa therapy.C. rf VIIa therapy, splenic artery rupture.D. Pelvic fracture, no TP.

78. Answer: BSplenic artery rupture is not a proven risk factor for throm-bosis (Answer A and Answer C are incorrect). Recombinant activated factor VII is a potential cause of venous throm-bosis (Answer B is correct). Although the presentation is several days after the administration of rf VIIa, deep venous thrombosis with secondary PE on day 5 cannot be excluded (Answer D is incorrect). 1. O’Connell KA, Wood JJ, Wise RP, Lozier JN, Braun MM.

Thromboembolic adverse events after use of recombinant human coagulation factor VIIa. JAMA 2006;295:293–8.

[PubMed Link]2. Hsia CC, Chin-Yee IH, McAlister VC. Use of recombinant

activated factor VII in patients without hemophilia. Ann Surg 2008;248:61–8.

[PubMed Link]

79. I.B. is a 56-year-old woman (65 kg) with a medical his-tory of many DVTs related to a protein S deficiency. She is in the ICU for streptococcal pneumonia with associated septic shock, necessitating intubation and mechanical ventilation. She is now recovering; her need for norepinephrine is decreasing, and her frac-tion of inspired oxygen has dropped from 80% to 50% in the past 48 hours. Her estimated CrCl is 40 mL/ minute. I.B. suddenly develops acute shortness of breath and tachycardia. Her systemic blood pressure is 120/80 mm Hg with a stable dose of norepinephrine. The medical team now suspects PE and asks for tho-racic angiography. Which treatment is best for I.B. while awaiting angiography?A. Initiate fondaparinux 5 mg subcutaneously

immediately.B. Give alteplase 100 mg, then initiate a heparin

80-international unit/kg bolus and an 18-unit/ kg/hour infusion.

C. Initiate enoxaparin 65 mg subcutaneously two times/day.

D. Delay treatment until angiography results are available.

79. Answer: CAnswer A is incorrect because the standard dose of fondaparinux for the treatment of PE is 7.5 mg once daily. Answer B is incorrect because this patient has no indication for thrombolytic therapy. Answer D is incorrect because prompt institution of therapy is essential. Initiate enoxapa-rin 65 mg subcutaneously two times/day is the best option listed (Answer C is correct).1. Warkentin TE, Greinacher A, Koster A, Lincoff AM. Treatment

and prevention of heparin-induced thrombocytopenia: American College of Chest Physicians Evidence-Based Clinical Practice Guidelines (8th ed). Chest 2008;133:340S–380S.

[PubMed Link]2. Torbicki A, Perrier A, Konstantinides S, Agnelli G, Galiè N,

Pruszczyk P, et al. Guidelines on the diagnosis and manage-ment of acute pulmonary embolism: the Task Force for the Diagnosis and Management of Acute Pulmonary Embolism of the European Society of Cardiology (ESC). Eur Heart J 2008;29:2276–315.

[PubMed Link]

80. M.W. is a 86 year-old man admitted for urosepsis. His medical history includes benign prostatic hyperplasia, chronic kidney failure, and diabetes. His current creati-nine clearance is estimated at 20 mL/minute. Which of the following TP regimens is most appropriate for M.W.?A. Enoxaparin 30 mg subcutaneously two times/day.B. Dalteparin 5000 units subcutaneously once daily.C. Dalteparin 2500 units subcutaneously once daily

PSAP-VII • Critical and Urgent Care36Answers

D. Fondaparinux 7.5 mg subcutaneously once daily.

80. Answer: BFondaparinux is contraindicated in patients with severe renal failure (Answer D is incorrect). Enoxaparin requires dosing adjustment in renal failure to 30 mg once daily (Answer A is incorrect). Finally, based on the recent DIRECT study, prophylactic dose dalteparin does not seem to require adjustments in renal failure (Answer C is incorrect; Answer B is correct). 1. Nutescu EA, Spinler SA, Wittkowsky A, Dager WE. Low-

molecular-weight heparins in renal impairment and obesity: available evidence and clinical practice recommendations across medical and surgical settings. Ann Pharmacother 2009;43:1064–83.

[PubMed Link]2. Douketis J, Cook D, Meade M, Guyatt G, Geerts W, Skrobik

Y. Prophylaxis against deep vein thrombosis in critically ill patients with severe renal insufficiency with the low-molec-ular-weight heparin dalteparin: an assessment of safety and pharmacodynamics: the DIRECT study. Arch Intern Med 2008;168:1805–12.

[PubMed Link]